Tải bản đầy đủ (.pdf) (51 trang)

 Tuyển tập Đề thi tuyển chọn đội tuyển dự thi VMO cả nước năm 2019

Bạn đang xem bản rút gọn của tài liệu. Xem và tải ngay bản đầy đủ của tài liệu tại đây (1.73 MB, 51 trang )

<span class='text_page_counter'>(1)</span><div class='page_container' data-page=1>

<b>ĐỀ THI CHÍNH THỨC </b>

NĂM 2018–2019


<b>Mơn: TỐN</b>


Thời gian:<b> 180 </b>phút (khơng kể thời gian giao đề)
Ngày thi: 15/9/2018


<b>Bài 1: </b><i>(5.0 điểm) </i>Giải phương trình 1 1


1
1


1 <sub>1</sub>


1


<i>x</i>


<i>x</i>


 









, trong đó vế trái gồm có


2018 dấu phân số.



<b>Bài 2: </b><i>(5.0 điểm) </i>Cho dãy số <sub>1</sub> 0, <sub>1</sub> 2 <sub>2</sub> 1


1
<i>n</i>


<i>n</i>


<i>x</i> <i>x</i> <i>n</i>


<i>x</i>


   


 . Chứng minh rằng dãy có


giới hạn hữu hạn và tính giới hạn đó.


<b>Bài 3: </b><i>(5.0 điểm) </i>Cho tứ giác <i>ABCD</i> khơng phải là hình thang, nội tiếp đường trịn


( )<i>O</i> . <i>AC</i> cắt <i>BD</i> tại <i>G</i>. <i>K</i> là điểm di động trên đoạn <i>OG</i>. Đường tròn ngoại
tiếp tam giác <i>KAD</i> và đường tròn ngoại tiếp tam giác <i>KBC</i> cắt nhau tại điểm
thứ hai <i>L</i>. Đường tròn ngoại tiếp tam giác <i>KAB</i> và đường tròn ngoại tiếp tam
giác <i>KCD</i> cắt nhau tại điểm thứ hai <i>I</i>. Chứng minh rằng khi <i>K</i> di động thì


đường thẳng <i>LI</i> luôn đi qua một điểm cố định hoặc song song với một đường


thẳng cố định.



<b>Bài 4: </b><i>(5.0 điểm) </i>Cho số nguyên dương <i>n</i>3,<i>n</i>6 và một bảng <i>n n</i> ô vuông, ban
đầu các ô trong bảng đều đánh số 0 . Mỗi lượt chơi, người chơi chọn một bảng
con gồm (<i>n</i>1) ( <i>n</i>1) ô vuông, sau đó cộng hoặc trừ tất cả các số trong bảng
con này với 1 (cùng một lượt chơi, có thể có số cộng 1, có số trừ 1). Liệu có thể
sau một số hữu hạn lượt chơi thì các ơ trong bảng chứa đầy đủ các số nguyên
dương từ 1 đến <i><sub>n</sub></i>2<sub> không? </sub>


SỞ GIÁO DỤC VÀ ĐÀO TẠO


HẢI PHỊNG



</div>
<span class='text_page_counter'>(2)</span><div class='page_container' data-page=2>

<b>HƯỚNG DẪN CHẤM </b>



<b>Bài </b> <b>Nội dung </b> <b>Điểm </b>


<b>1. </b>


<i>(5.0đ) </i>


Khi vế trái gồm có <i>n</i> dấu phân số và ký hiệu là <i>S<sub>n</sub></i>, ta chứng minh


bằng quy nạp rằng 1


1


<i>n</i> <i>n</i>


<i>n</i>


<i>n</i> <i>n</i>



<i>F x</i> <i>F</i>
<i>S</i>


<i>F x</i> <i>F</i>






 , trong đó

 

<i>Fn</i> là dãy số


Fibonacci: <i>F</i><sub>0</sub> 0,<i>F</i><sub>1</sub>1,<i>F<sub>n</sub></i><sub></sub><sub>1</sub> <i>F<sub>n</sub></i> <i>F<sub>n</sub></i><sub></sub><sub>1</sub>.


Rõ ràng <i>S</i><sub>1</sub> 1 1 <i>x</i> 1


<i>x</i> <i>x</i>




   (thỏa mãn).


1.0


Nếu ta đã có 1


1


<i>n</i> <i>n</i>



<i>n</i>


<i>n</i> <i>n</i>


<i>F x</i> <i>F</i>
<i>S</i>


<i>F x</i> <i>F</i>






 thì  <sub></sub>



   


1
1
1
1 1
1 1
<i>n</i>
<i>n</i> <i>n</i>
<i>n</i>
<i>n</i> <i>n</i>
<i>S</i>



<i>F x</i> <i>F</i>
<i>S</i>


<i>F x</i> <i>F</i>



 

1
1


1 <i>n</i> <i>n</i>


<i>n</i> <i>n</i>


<i>F x</i> <i>F</i>
<i>F x</i> <i>F</i>


 




2 1
1
<i>n</i> <i>n</i>
<i>n</i> <i>n</i>



<i>F x</i> <i>F</i>
<i>F x</i> <i>F</i>


2.0


Phương trình trở thành


2
2019 2018


2018 2018 2018
2018 2017


0


<i>F</i> <i>x</i> <i>F</i>


<i>x</i> <i>F</i> <i>x</i> <i>F</i> <i>x</i> <i>F</i>


<i>F</i> <i>x</i> <i>F</i>


    




2 1 5


1 0



2


<i>x</i> <i>x</i> <i>x</i> 


      (đpcm)


2.0


<b>2. </b>


<i>(5.0đ) </i>


Rõ ràng ta có <i>x<sub>n</sub></i><sub></sub>0; 2<sub></sub>  <i>n</i> 1.


Ta có <sub>1</sub> 0, <sub>2</sub> 2, <sub>3</sub> 2, <sub>4</sub> 50


5 29


<i>x</i>  <i>x</i>  <i>x</i>  <i>x</i>  .


Xét hàm số

 

2 <sub>2</sub>


1


<i>f x</i>


<i>x</i>


 nghịch biến trên đoạn 0; 2 .



1.5


Khi đó hàm số <i>f f x</i>

 

đồng biến trên đoạn 0; 2<sub></sub> <sub></sub>.
Do <i>x</i><sub>1</sub><i>x</i><sub>3</sub>  <i>f f x</i>

 

<sub>1</sub>

 <i>f f x</i>

 

<sub>3</sub>

<i>x</i><sub>3</sub> <i>x</i><sub>5</sub>


 



3

 

5

5 7...


<i>f f x</i> <i>f f x</i> <i>x</i> <i>x</i>


   


Hay là

<i>x</i><sub>2</sub><i><sub>n</sub></i><sub>1</sub>

là dãy số tăng, bị chặn trên bởi 2 nên tồn tại giới


hạn hữu hạn lim<i>x</i><sub>2</sub><i><sub>n</sub></i><sub></sub><sub>1</sub><sub>  </sub><i>a</i> 0; 2<sub></sub>.


</div>
<span class='text_page_counter'>(3)</span><div class='page_container' data-page=3>

giới hạn hữu hạn lim<i>x</i><sub>2</sub><i><sub>n</sub></i> <sub>  </sub><i>b</i> 0; 2<sub></sub>.


Ta có hệ





2
2


2
2


2



2


1 <sub>1</sub> <sub>0</sub>


2 2


1


<i>a</i> <i><sub>a ab</sub></i>


<i>b</i> <i><sub>a b</sub></i> <i><sub>ab</sub></i>


<i>b</i> <i>a b</i>
<i>b</i>


<i>a</i>




 <sub> </sub> <sub></sub>


  <sub></sub> <sub></sub> <sub></sub> <sub></sub> <sub></sub>


 


 





 <sub></sub> 


 




- Nếu <i><sub>a</sub></i><sub></sub><i><sub>b</sub></i><sub></sub><i><sub>a</sub></i>3<sub></sub><i><sub>a</sub></i><sub></sub><sub>2</sub><sub></sub><sub>0</sub><sub></sub><i><sub>a</sub></i><sub></sub><i><sub>b</sub></i><sub></sub><sub>1</sub><sub>. </sub>


- Nếu <i><sub>ab</sub></i><sub></sub><sub>1</sub><sub></sub><i><sub>a</sub></i>2<sub></sub><sub>2</sub><i><sub>a</sub></i><sub></sub><sub>1</sub><sub></sub><sub>0</sub><sub></sub><i><sub>a</sub></i><sub></sub><i><sub>b</sub></i><sub></sub><sub>1</sub><sub>. </sub>


Vậy tồn tại lim<i>x<sub>n</sub></i> 1.


2.0


<b>3. </b>


<i>(5.0 đ) </i>


Gọi M là giao điểm của AD và BC. N là giao điểm của AB và CD.
Khi đó OG vng góc với MN tại H (H là điểm Miquel của tứ
giác ABCD).


1.5


Tứ giác HADN nội tiếp nên <i>MH MN</i>. <i>MA MD</i>. .


Tứ giác ALKD nội tiếp nên <i>ML MK</i>. <i>MA MD</i>. .


Suy ra <i>MH MN</i>. <i>ML MK</i>. , do đó tứ giác NHLK nội tiếp.



1.5


  <sub>90</sub>0


<i>NLK</i> <i>NHK</i> <i>NL</i> <i>MK</i>


     0.5


Tương tự ta có <i>NL</i><i>MK MI</i>, <i>NK KH</i>, <i>MN</i> 3 đường thẳng


NL, MI, KH đồng quy. Khi đó hoặc LI // MN cố định hoặc LI cắt
MN tại J thì

<i>MNHJ</i>

 1.


Do M, N, H cố định nên J cố định.


1.5


</div>
<span class='text_page_counter'>(4)</span><div class='page_container' data-page=4>

<i>(5.0 đ) </i> Gọi , , ,<i>a b c d</i> là số lần mà bảng con

<i>n</i>1

 

 <i>n</i>1

ở góc trái trên,
phải trên, phải dưới và trái dưới được chọn sau tất cả các lượt
chơi. Khi đó các ơ ở góc trái trên, phải trên, phải dưới và trái dưới
sẽ đánh số , , ,<i>a b c d</i>; <i>n</i>2 ơ ở dịng 1 mà khơng phải ơ góc sẽ
đánh số <i>a</i><i>b</i>; <i>n</i>2 ô ở cột 1 mà không phải ô góc sẽ đánh số


<i>a</i><i>d</i>; <i>n</i>2 ơ ở cột <i>n</i> mà khơng phải ơ góc sẽ đánh số <i>b</i><i>c</i>; <i>n</i>2
ơ ở dịng <i>n</i> mà khơng phải ơ góc sẽ đánh số <i>c</i><i>d</i>;

<i>n</i>2

2 ơ cịn
lại ở bảng con

<i>n</i>2

 

 <i>n</i>2

chính giữa sẽ đánh số <i>a</i><i>b</i><i>c</i><i>d</i>


.


Trong tập các số từ 1 đến <i><sub>n</sub></i>2<sub> thì số số lẻ bằng hoặc nhiều hơn số </sub>



số chẵn là 1 . 0.5


– Nếu <i>a</i><i>b</i><i>c</i><i>d</i>

mod 2

thì trong bảng có nhiều nhất 4 ơ lẻ
và ít nhất 2


4


<i>n</i>  ô chẵn, khi đó

2

2

<sub> </sub>



4 <i>n</i> 4 8<i>n</i>  0; 1 . 0.5


– Nếu <i>a</i><i>b</i><i>c</i>0 mod 2 ,

<i>d</i>1 mod 2

thì trong bảng có


<i>n</i>1

2 ơ lẻ và 2<i>n</i>1 ơ chẵn, khi đó


<i><sub>n</sub></i><sub></sub><sub>1</sub>

2<sub></sub>

<sub>2</sub><i><sub>n</sub></i><sub></sub><sub>1</sub>

<sub></sub><i><sub>n</sub></i>2<sub></sub><sub>4</sub><i><sub>n</sub></i><sub></sub><sub>2</sub><sub></sub>

 

<sub>0; 1</sub> <sub>. </sub>


0.5


– Nếu <i>a</i><i>b</i><i>c</i>1 mod 2 ,

<i>d</i>0 mod 2

thì trong bảng có


<i>n</i>1

22 ơ lẻ và 2<i>n</i>3 ơ chẵn, khi đó


2

<sub>2</sub>

 



1 2 2 3 4 6 0; 1


<i>n</i>   <i>n</i> <i>n</i>  <i>n</i>  .



0.5


– Nếu <i>a</i><i>b</i>1 mod 2 ,

<i>c</i><i>d</i>0 mod 2

thì trong bảng số ô


lẻ ít hơn số ô chẵn là

<i>n</i>2

2. 0.5


– Nếu <i>a</i><i>c</i>0 mod 2 ,

<i>b</i><i>d</i>1 mod 2

thì trong bảng có




4 <i>n</i>2 24<i>n</i>6 ơ lẻ và <i><sub>n</sub></i>2<sub></sub><sub>4</sub><i><sub>n</sub></i><sub></sub><sub>6</sub><sub> ô chẵn, khi đó </sub>

<sub>4</sub><i><sub>n</sub></i><sub></sub><sub>6</sub>

<sub></sub>

<i><sub>n</sub></i>2 <sub></sub><sub>4</sub><i><sub>n</sub></i><sub></sub><sub>6</sub>

<sub> </sub><i><sub>n</sub></i>2<sub></sub><sub>8</sub><i><sub>n</sub></i><sub></sub><sub>12</sub><sub></sub>

 

<sub>0; 1</sub> <sub>. </sub>


Vậy các ô trong bảng không thể chứa đầy đủ các số từ 1 đến <i><sub>n</sub></i>2


.


0.5


</div>
<span class='text_page_counter'>(5)</span><div class='page_container' data-page=5>

SỞ GIÁO DỤC VÀ ĐÀO TẠO


HẢI PHỊNG



<b>ĐỀ THI CHÍNH THỨC </b>


KỲ THI CHỌN HỌC SINH GIỎI BẢNG A VAØ


ĐỘI TUYỂN DỰ THI HỌC SINH GIỎI QUỐC GIA



NĂM 2018–2019



<b>Mơn: TỐN</b>



Thời gian:<b> 180 </b>phút (không kể thời gian giao đề)
Ngày thi: 16/9/2018


<b>Bài 1: </b><i>(5.0 điểm) </i>Cho tam giác <i>ABC</i> nhọn nội tiếp đường tròn

 

<i>O</i> , <i>AB</i><i>AC</i>.


<i>M</i> là điểm chính giữa của cung trịn <i>BC</i> khơng chứa <i>A</i>. <i>J</i> là trung điểm <i>BC</i>.


<i>K</i> là hình chiếu vng góc của <i>M</i> trên <i>CA</i>. <i>BK</i> cắt <i>AJ</i> tại <i>X</i>. <i>CX</i> cắt <i>JK</i> tại


<i>L</i>. Đường thẳng qua <i>J</i>và vuông góc với <i>MK</i>, cắt <i>AL</i> tại <i>T</i>. <i>JK</i> cắt <i>AB</i> tại <i>I</i>.


Chứng minh rằng <i>CT</i>vng góc với <i>IM</i>.


<b>Bài 2: </b><i>(5.0 điểm) </i>Có bao nhiêu số nguyên dương <i>n</i> không vượt quá 2019 mà <sub>2</sub><i>n</i>


<i>n</i>


<i>C</i>


chia hết cho 19 ?


<b>Bài 3: </b><i>(5.0 điểm) </i>Cho <i>P x</i>

 

là đa thức có bậc khơng vượt quá 2017 thỏa mãn


 

1

0; 1; ...; 2017



<i>P k</i> <i>k</i>  <i>k</i> , chứng minh rằng


<sub>2018</sub>

 

<sub>1</sub> <sub>2019 2</sub>

2018 <sub>1</sub>




<i>P</i>  <i>P</i>    .


<b>Bài 4: </b><i>(5.0 điểm) </i>Với <i>n</i>2, gọi <i>f n</i>

 

là số các hoán vị của tập

1; 2; 3;...;<i>n</i>


khơng có số <i>k</i> nào đứng liền trước số <i>k</i>1 với mọi <i>k</i>

1; 2; 3;...;<i>n</i>1

. Chứng
minh rằng:


a) <i>f n</i>

  

 <i>n</i>1

 

<i>f n</i>1

 

 <i>n</i>2

 

<i>f n</i>2

 <i>n</i> 4.


b)

 

1

1 !

1 2


2


<i>n</i>


<i>f n</i> <i>n</i>


<i>n</i> <i>e</i>


  


     


 


 


, trong đó  <sub> </sub><i>x</i> là phần nguyên của <i>x</i> và


1
lim 1



<i>n</i>


<i>e</i>


<i>n</i>


 


 <sub></sub>  <sub></sub>


  .


</div>
<span class='text_page_counter'>(6)</span><div class='page_container' data-page=6>

<b>HƯỚNG DẪN CHẤM </b>



<b>Bài</b> <b>Nội dung</b> <b>Điểm</b>


<b>1. </b>


<i>(5.0đ) </i> Gọi <i>S</i> là giao điểm của <i>AL</i> và <i>BC</i>.


Xét tam giác <i>AJC</i> với các đường đồng quy <i>AS JK CX</i>, , và các điểm
thẳng hàng , ,<i>B X K</i> ta có

<i>J C B S</i>, , ,

 1.


2.0


Hay là <i>SC</i> <i>BC</i> <i>BC SC</i> <i>BC SC</i> <i>SB</i>


<i>SJ</i> <i>BJ</i> <i>BJ SJ</i> <i>CJ SJ</i> <i>SC</i>



 


   


  . 1.0


Mà <i>JT</i> // <i>AC</i> (cùng vng góc với <i>MK</i>) nên suy ra
//


<i>SB</i> <i>SC</i> <i>SA</i>


<i>AB</i> <i>CT</i>


<i>SC</i>  <i>SJ</i>  <i>ST</i>  .


1.0


Lại có , ,<i>I J K</i> là đường thẳng Simson của tam giác <i>ABC</i> nên


<i>MI</i> <i>AB</i>, từ đó ta có <i>MI</i><i>CT</i>. 1.0


<b>2. </b>


<i>(5.0đ) </i>


Ta có:

 



 

 

 



 



 


  


 


 


19 2 19 2 19 19


2 !


2 ! 2 !


!
<i>n</i>


<i>n</i>


<i>n</i>


<i>v</i> <i>C</i> <i>v</i> <i>v</i> <i>n</i> <i>v</i> <i>n</i>


<i>n</i>





   



 <sub></sub><sub></sub> <sub></sub> <sub></sub> <sub></sub><sub></sub>


   


 



1


2
2


19<i>k</i> 19<i>k</i>


<i>k</i>


<i>n</i> <i>n</i>


1.0


Mà: 2<sub></sub> <i>x</i><sub></sub>2 <sub> </sub><i>x</i> , đẳng thức xảy ra khi và chỉ khi 0

 

1
2


<i>x</i>


  . 1.0


Khi đó: <sub>2</sub><i>n</i> 



<i>n</i>


<i>C</i> 

 <sub></sub> <sub></sub> <sub></sub> <sub></sub>  


   


19 2


2


19 0 2 0 1


19 19


<i>n</i>


<i>n</i> <i>k</i> <i>k</i>


<i>n</i> <i>n</i>


<i>v</i> <i>C</i> <i>k</i>


 


<sub></sub> <sub></sub>  


 


1



1
2


19<i>k</i>


<i>n</i>


<i>k</i> .


</div>
<span class='text_page_counter'>(7)</span><div class='page_container' data-page=7>

Phân tích <i>n</i> theo hệ cơ số 19:
0
.19
<i>m</i>
<i>i</i>
<i>i</i>
<i>i</i>
<i>n</i> <i>a</i>


<sub></sub>

, 0<i>a<sub>i</sub></i> 18 <i>i</i> 0,<i>m</i>, khi


đó <sub></sub> <sub></sub>  
 
1
1
2
19<i>k</i>
<i>n</i>
<i>k</i>
 


 
    
 
<sub></sub> <sub></sub>  
 
 
1 2


1.19 2.19 ... 0 1 <sub>1</sub>


2
19


<i>k</i> <i>k</i>


<i>k</i> <i>k</i>


<i>k</i>


<i>a</i> <i>a</i> <i>a</i>


<i>k</i> <i>a<sub>i</sub></i> 9  <i>i</i> 0


1.0


Và <sub>2019</sub><sub></sub><sub>5.19</sub>2<sub></sub><sub>11.19</sub><sub></sub><sub>5</sub><sub> nên số số </sub><i><sub>n</sub></i><sub> không vượt quá 2019 mà </sub>
2


<i>n</i>
<i>n</i>



<i>C</i>  19 là 6 10 10 1   599 số (0<i>a</i><sub>2</sub> 5, 0<i>a a</i><sub>1</sub>, <sub>0</sub>9, ngồi
ra khơng tính số 0 ).


Vậy số số <i>n</i> cần tìm là 2019 599 1420.


1.0


<b>3. </b>


<i>(5.0đ)</i>


Theo công thức nội suy Lagrange cho <i>P x</i>

 

với các số


0,1, 2,..., 2017 ta có

 



 


 


2017
2017
2017
0 ,
0
1


! 2017 !


<i>i</i>
<i>j</i> <i>j i</i>
<i>i</i>



<i>x</i> <i>j</i>


<i>P x</i> <i>P i</i>


<i>i</i> <i>i</i>

 

 




1.0
Khi đó

 

 




 
 
 
 


2017
2017 2017
0 0
1 <sub>1</sub>


2018 2018! 2018!



! 2018 ! ! 2018 !


<i>i</i>


<i>i</i> <i>i</i>


<i>P i</i> <i><sub>i</sub></i>


<i>P</i>


<i>i</i> <i>i</i> <i>i</i> <i>i</i>




<sub></sub>


2017
2018
0
1 <i>i</i>
<i>i</i>
<i>i</i> <i>C</i>
1.0

 

 

 

 


 


 





 


  
  

  
 



2017
2017
2018
0
2017 2017
1
2018
0 0
1


1 1 2018!


1 ! 2017 !


1


2018! 1 .


1 ! 2017 !


<i>i</i>
<i>i</i>


<i>i</i>
<i>i</i> <i>i</i>
<i>P i</i>
<i>P</i>
<i>i</i> <i>i</i>
<i>i</i>
<i>i</i> <i>C</i>
<i>i</i> <i>i</i>
1.0
Từ đó:


 

2017

<sub></sub>

1

<sub></sub>

2017

1


2018 2018 2019


0 0


2018 1 1 <i>i</i> <i>i</i> 1 <i>i</i>


<i>i</i> <i>i</i>


<i>P</i> <i>P</i> <i>i</i> <i>C</i> <i>C</i> <i>i</i> <i>C</i>


 


  

<sub></sub>

  

<sub></sub>



(Do 1 1


2018 2018 2019



<i>i</i> <i>i</i> <i>i</i>


<i>C</i> <i>C</i> <i>C</i>


  )
1.0


2017
2018
2018
0


2019 <i>i</i> 2019 2 1


<i>i</i>


<i>C</i>


<sub></sub>

 


(Do

<i><sub>i</sub></i> 1

<i><sub>C</sub></i><sub>2019</sub><i>i</i>1 2019<i><sub>C</sub></i><sub>2018</sub><i>i</i>


  )


</div>
<span class='text_page_counter'>(8)</span><div class='page_container' data-page=8>

<b>4. </b>


<i>(5.0đ)</i>



a



<i>(2.0đ)</i>


– Xét một hoán vị bất kỳ của

1; 2; 3;...;<i>n</i>

thỏa mãn đề bài, và:
+) Nếu không tồn tại <i>k</i> để , ,<i>k n k</i>1 liên tiếp theo thứ tự đó


thì có thể bỏ <i>n</i> đi để được một hoán vị của


1; 2; 3;...;<i>n</i>1

thỏa mãn.


0.5


+ Nếu tồn tại <i>k</i> để , ,<i>k n k</i>1 liên tiếp theo thứ tự đó thì có
thể bỏ cả <i>n</i> và <i>k</i>1 đi, đồng thời giảm đi 1 ở tất cả các
số từ <i>k</i>2 đến <i>n</i>1 (<i>k</i><i>n</i>2 thì khơng cần làm bước
này), khi đó sẽ được một hốn vị của

1; 2; 3;...;<i>n</i>2


thỏa mãn.


0.5


– Ngược lại:


+ Từ một hoán vị của

1; 2; 3;...;<i>n</i>1

thỏa mãn thì có thể
tạo ra <i>n</i>1 hoán vị của

1; 2; 3;...;<i>n</i>

thỏa mãn bằng cách
chèn <i>n</i> vào vị trí đầu tiên hoặc <i>n</i>2 vị trí liền sau các số
1, 2,...,<i>n</i>2. Sẽ có tổng cộng

<i>n</i>1

 

<i>f n</i>1

hoán vị
dạng này.


0.5



+ Từ một hoán vị của

1; 2; 3;...;<i>n</i>2

thỏa mãn thì có thể
tạo ra <i>n</i>2 hốn vị của

1; 2; 3;...;<i>n</i>

thỏa mãn bằng cách:
với mỗi <i>k</i> 1,<i>n</i>2, tăng thêm 1 ở tất cả các số từ <i>k</i>1
đến <i>n</i>2 (<i>k</i><i>n</i>2 thì khơng cần làm bước này), sau đó
chèn ,<i>n k</i>1 vào vị trí liền sau <i>k</i>. Sẽ có tổng cộng


<i>n</i>2

 

<i>f n</i>2

hốn vị dạng này.


Vậy <i>f n</i>

  

 <i>n</i>1

 

<i>f n</i>1

 

 <i>n</i>2

 

<i>f n</i>2

 <i>n</i> 4.


0.5


 








 


 

<sub></sub>



0


1 ! 1


1 !
!


<i>k</i>



<i>k</i>


<i>n</i>


<i>n</i>


<i>e</i> <i>k</i>


 



 



  


 


 

<sub></sub>

 

<sub></sub>



1


0 2


1 1


1 ! 1 !


! !


<i>k</i> <i>k</i>


<i>n</i>



<i>k</i> <i>k n</i>


<i>n</i> <i>n</i>


<i>k</i> <i>k</i> ,


</div>
<span class='text_page_counter'>(9)</span><div class='page_container' data-page=9>

trong đó

 


1
0
1
1 !
!
<i>k</i>
<i>n</i>
<i>k</i>
<i>n</i>
<i>k</i>




<sub></sub>

là số nguyên.


<b>b/ </b>


<i>(2.0đ)</i> Và

 




  

 
     

   


1


2 2 2


1 1 1


1 ! 1 !


! ! <sub>2</sub>


<i>k</i>


<i>k n</i>


<i>k n</i> <i>k n</i> <i>k n</i>


<i>n</i> <i>n</i>


<i>k</i> <i>k</i> <i><sub>n</sub></i>


 

1 1
1 2


<i>n</i> .
0.5


Do đó

 



1
0


1 ! <sub>1</sub> 1


1 !
2 !
<i>k</i>
<i>n</i>
<i>k</i>
<i>n</i>
<i>n</i>
<i>e</i> <i>k</i>


   
  
 
 
 



. 0.5


Ta chứng minh khẳng định đề bài bằng quy nạp.
Rõ ràng kết quả đã đúng với 2, 3 .



Nếu đã có <i>f n</i>

1 ,

 

<i>f n</i>2

, ta cần chứng minh


  

 

  



1 1


0 0 0


1 1 ! 1 ! 1 1 !


1


! ! !


<i>k</i> <i>k</i> <i>k</i>


<i>n</i> <i>n</i> <i>n</i>


<i>k</i> <i>k</i> <i>k</i>


<i>n</i> <i>n</i> <i>n</i>


<i>n</i> <i>k</i> <i>k</i> <i>k</i>


 
  
    
 



0.5


 

1

  

1

  



0


1 1 1 1 1 1 !


!


<i>n</i> <i>n</i> <i>k</i>


<i>n</i>


<i>k</i>


<i>n</i> <i>n</i>


<i>n</i> <i>n</i> <i>n</i> <i>k</i>








    


  

<sub></sub>




 

1

 

  



0


1 ! 1 1 !


1
! !
<i>k</i> <i>k</i>
<i>n</i>
<i>n</i>
<i>k</i>
<i>n</i> <i>n</i>
<i>k</i> <i>k</i>


 <sub></sub> <sub></sub> <sub></sub> 
 
   
 
 


 

  



 
  

<sub></sub>


1
0


1 1 !



1
1
!
<i>k</i>
<i>n</i>
<i>n</i>
<i>k</i>
<i>n</i>
<i>n</i> <i>k</i>

 

  





  
  

<sub></sub>


1
0


1 1 1 !


1
!
<i>k</i>
<i>n</i>
<i>n</i>
<i>k</i>
<i>n</i> <i>n</i>


<i>k</i> (đúng).


1.0



–––––––––– HẾT ––––––––––


<i><b>Lưu ý khi chấm bài: </b></i>


<i>– Đáp án chỉ trình bày một cách giải bao gồm các ý bắt buộc phải có trong bài làm của </i>
<i>học sinh. Khi chấm nếu học sinh bỏ qua bước nào thì khơng cho điểm bước đó. </i>
<i>– Nếu học sinh giải cách khác, giám khảo căn cứ các ý trong đáp án để cho điểm. </i>
<i>– Trong bài làm, nếu ở một bước nào đó bị sai thì các phần sau có sử dụng kết quả sai </i>


<i>đó khơng được điểm. </i>


</div>
<span class='text_page_counter'>(10)</span><div class='page_container' data-page=10>

SỞ GIÁO DỤC VAØ ĐAØO TẠO


HƯNG YÊN



<b>ĐỀ THI CHÍNH THỨC </b>


KỲ THI CHỌN ĐỘI TUYỂN DỰ THI HỌC SINH


GIỎI CẤP QUỐC GIA NĂM 2019



<b>Mơn: TỐN</b>, bài thi thứ nhất


Thời gian:<b> 180 </b>phút (không kể thời gian giao đề)
Ngày thi: 28/08/2018


<b>Câu 1.</b><i>(4,0 điểm) </i>Cho , ,<i>a b c</i>là các số thực dương và <i>n</i>,<i>n</i>2. Tìm giá trị nhỏ
nhất của biểu thức


2 2 2



( ) ( ) ( )


<i>n</i> <i>a</i> <i>bc</i> <i>n</i> <i>b</i> <i>ac</i> <i>n</i> <i>c</i> <i>ab</i>


<i>P</i>


<i>a b c</i> <i>b a c</i> <i>c a b</i>


  


  


   .


<b>Câu 2.</b><i>(4,0 điểm) </i>Cho <i>a</i> và <i>b</i> là các số nguyên dương. Xét dãy số ( )<i>u<sub>n</sub></i> xác định bởi


2 2


,


<i>n</i> <i>a n</i>


<i>u</i>  <i>bn</i> với <i>n</i>1; 2; .... Tính lim

 

<i><sub>n</sub></i>


<i>n</i> <i>u</i> , trong đó kí hiệu

 

<i>x</i> là phần lẻ


của số thực .<i>x</i>


<b>Câu 3.</b><i>(4,0 điểm) </i>Cho tam giác <i>ABC</i> nhọn, không cân nội tiếp đường tròn

 

<i>O</i> .



Gọi <i>I</i> là tâm đường tròn nội tiếp tam giác<i>ABC</i>. Điểm <i>E</i> đối xứng với <i>B</i> qua


<i>IC</i>, <i>F</i> đối xứng với <i>C</i> qua <i>IB</i>. Gọi <i>M</i> là trung điểm của cung <i>BAC</i> của đường
tròn

 

<i>O</i> . Đường thẳng <i>MI</i> cắt đường tròn ngoại tiếp tam giác <i>BIC</i> tại <i>N</i> (khác


<i>I</i>). Đường thẳng <i>AI</i> cắt

 

<i>O</i> tại điểm thứ hai <i>D</i>(khác <i>A</i>), đường thẳng qua <i>I</i>


vng góc với <i>AD</i> cắt <i>BC</i> tại <i>K</i>. Chứng minh rằng
a) <i>EF</i><i>OI</i>.


b) <i>KN</i> là tiếp tuyến của đường tròn ngoại tiếp tam giác<i>BIC</i>.
<b> Câu 4.</b><i>(4,0 điểm) </i>Cho các số nguyên dương<i>a a</i><sub>1</sub>, <sub>2</sub>, ...,<i>a</i><sub>2018</sub>. Xét tập


 

 

 



1 2 2018



2018 1 1 1 2 ... 1 2018; , 1,..., 2018 .


<i>k</i> <i>k</i> <i>k</i>


<i>i</i>


<i>S</i> <sub></sub> <sub></sub> <i>a</i> <sub> </sub> <i>a</i> <sub></sub> <sub> </sub> <i>a</i> <i>k</i> <sub></sub><sub></sub> <sub> </sub><i>i</i>


Chứng minh rằng tích của tất cả các phần tử thuộc <i>S</i><sub>2018</sub>là một số chính phương.


<b>Câu V</b><i>(4,0 điểm) </i>Cho đa giác lồi <i>P</i>. Bạn An muốn ghi vào mỗi đỉnh của <i>P</i> một số


nguyên dương sao cho các điều kiện sau được đồng thời thỏa mãn:


i) Trong các số được ghi, có ít nhất một số chẵn;


ii) Tổng của ba số được ghi ở ba đỉnh liên tiếp tùy ý là một số lẻ.


Chứng minh rằng bạn An có thể thực hiện được cách ghi như trên khi và chỉ
khi số đỉnh của <i>P</i> chia hết cho 3.


</div>
<span class='text_page_counter'>(11)</span><div class='page_container' data-page=11>

<b>ĐÁP ÁN VÀ BIỂU ĐIỂM </b>



<b>Câu </b> <b>Nội dung </b> <b>Điểm </b>


<b>1. </b>
(<i>4 đ</i>)


Cho , ,<i>a b c</i>là các số thực dương và <i>n</i>,<i>n</i>2. Tìm giá trị nhỏ nhất
của biểu thức


2 2 2


( ) ( ) ( )


<i>n</i> <i>a</i> <i>bc</i> <i>n</i> <i>b</i> <i>ac</i> <i>n</i> <i>c</i> <i>ab</i>


<i>P</i>


<i>a b c</i> <i>b a c</i> <i>c a b</i>


  


  



  


Áp dụng bất đẳng thức <i>AM</i><i>GM</i> ta có


  




  


2 2 2


3


3. . .


( ) ( ) ( )


<i>n</i> <i>a</i> <i>bc b</i> <i>ac c</i> <i>ab</i>


<i>P</i>


<i>a b c b a c c a b</i> (1).


Ta chứng minh    


  


2 2 2



. . 1


( ) ( ) ( )


<i>a</i> <i>bc b</i> <i>ac c</i> <i>ab</i>


<i>a b c b a c c a b</i> (2).


2,0 đ


Thật vậy: Do , ,<i>a b c</i> dương nên bất đẳng thức (1) đưa về

<i>a</i>2<i>bc b</i>



2<i>ac c</i>



2 <i>ab</i>

<i>abc a b b c c</i>





<i>a</i>

(3).
Áp dụng bất đẳng thức Cauchy – Schwarz cho 2 bộ 2 số

<i>a</i>, <i>bc</i>



<i>b</i>, <i>bc</i>

ta được

<i><sub>a</sub></i>2<sub></sub><i><sub>bc b</sub></i>



2<sub></sub><i><sub>bc</sub></i>

<sub></sub>

<i><sub>ab bc</sub></i><sub></sub>

2<sub> </sub>


hay <i>b a</i>

2<i>bc b c</i>

<i>b a c</i>2

2 (4)


Chứng minh tương tự ta có


2

 2

2


<i>c b</i> <i>ac a c</i> <i>c a b</i> (5)


2

 2

2


<i>a c</i> <i>ab a b</i> <i>a c b</i> (6)


Vì các vế của (4), (5), (6) đều dương. Nhân vế với vế các bất đẳng


thức đó với nhau, rồi chia cả 2 vế của bất đẳng thức thu được cho








<i>abc a b b c c a</i>   ta được (3), do đó có (2), suy ra <i>P</i>3. Dấu
bằng xảy ra khi và chỉ khi dấu bằng xảy ra đồng thời ở (1), (4), (5)
và (6), khi đó <i>a</i><i>b</i><i>c</i>. Vậy giá trị nhỏ nhất của <i>P</i> là 3.


2,0 đ


<b>2. </b>
(<i>4 đ</i>)


Cho <i>a</i> và <i>b</i> là các số nguyên dương. Xét dãy số ( )<i>u<sub>n</sub></i> xác định bởi


2 2 <sub>,</sub>


<i>n</i> <i>a n</i>


<i>u</i>  <i>bn</i> với <i>n</i>1; 2; .... Tính lim

 

<i><sub>n</sub></i>


<i>n</i> <i>u</i> , trong đó kí hiệu

 

<i>x</i>


là phần lẻ của số thực .<i>x</i>


Lấy <i>b</i> chia cho 2 ,<i>a</i> ta được <i>b</i>2<i>aq</i><i>r q</i>; ,<i>r</i>;0<i>r</i>2 .<i>a</i> Xảy ra 2
trường hợp sau


<i><b>Trường hợp 1.</b></i> Nếu <i>r</i>0, thì <i>b</i>2<i>aq</i> và



2 2 2 2 2


2 ( ) ( ) .


<i>n</i> <i>a n</i> <i>aqn</i> <i>an q</i> <i>q</i> <i>an q</i>


<i>u</i>       


</div>
<span class='text_page_counter'>(12)</span><div class='page_container' data-page=12>

Lại có 2 2


( 1) 2 ( 1) .


<i>n</i> <i>an q</i> <i>an</i>


<i>u</i>      <i>q</i>


Từ đây suy ra


2
2


( 1) ; ( 1) .


2
<i>n</i>


<i>u</i> <i>n</i>


<i>a</i>
<i>an</i> <i>q</i>   <i>q</i>



  


Kết hợp lại ta được


2
2 <sub>(</sub> <sub>)</sub>2<sub>;</sub> ( 1) <sub>.</sub>


( 1)


2
<i>n</i>


<i>u</i> <i>an</i> <i>q</i>


<i>an q</i> <i>q</i> <i>n</i>


<i>a</i>


   


    Điều


này dẫn tới


2
( 1)
1; .
2
<i>n</i>


<i>q</i>


<i>u</i> <i>an q</i> <i>n</i>


<i>a</i>


 <sub> </sub> <sub></sub> <sub></sub> <sub> </sub>


  <b> </b>


Khi đó


 

2 2



lim lim lim 2 ( 1)


<i>n</i>
<i>n</i>
<i>n</i>


<i>n</i> <i>n</i>


<i>n</i> <i>u</i>  <i>u</i> <i>u</i>  <i>a</i> <i>n</i> <i>aqn</i> <i>an</i> <i>q</i>


 
      
 
2
2


2 2
2
( 1)
2


2 ( 1)


li


2


m lim 1.


2 1


1


<i>n</i> <i>n</i>


<i>q</i>
<i>a</i>


<i>an</i> <i>q</i> <i><sub>n</sub></i>


<i>aq</i> <i>q</i>


<i>a</i> <i>n</i> <i>aqn</i> <i>an</i> <i>q</i> <i><sub>a</sub></i> <i><sub>a</sub></i>


<i>n</i> <i>n</i>
 




 
  

  

 

<i><b>Trường hợp 2.</b> </i>


Nếu 0 <i>r</i> 2 ,<i>a</i> thì từ 2 2


( 1) ( 2 ) ( 1) 0


<i>n</i> <i>an q</i> <i>r</i> <i>a n</i> <i>q</i>


<i>u</i>         và


2 2


( ) ,


<i>n</i>


<i>u</i>  <i>an q</i> <i>rn q</i> ta suy ra


2


2 <sub>(</sub> <sub>1</sub>2



) ) ;


(<i>an q</i> <i>u<sub>n</sub></i> <i>an</i> <i>n</i> <i>q</i>


<i>r</i>
<i>q</i>


       hay


2


; .


<i>n</i>


<i>q</i>


<i>u</i> <i>an q</i> <i>n</i>


<i>r</i>


 <sub> </sub> <sub></sub> <sub> </sub>


 


Từ đây ta có


 

2 2




lim lim lim (2 )


<i>n</i> <i>un</i> <i>n</i> <i>un</i> <i>un</i> <i>n</i> <i>a</i> <i>n</i> <i>aq r n</i> <i>an q</i>


 
      
 
2
2
2 2
2
lim li


(2 ) m 2


<i>n</i> <i><sub>n</sub></i> <i><sub>aq</sub></i> <i><sub>r n</sub></i> <i>n</i>


<i>q</i>
<i>r</i>


<i>rn q</i> <i><sub>n</sub></i>


<i>aq</i> <i>r</i> <i>q</i>


<i>a</i> <i>an q</i> <i><sub>a</sub></i> <i><sub>a</sub></i>


<i>n</i> <i>n</i>
 



 

 

 
 
.
2 2
<i>r</i> <i>b</i>
<i>a</i> <i>a</i>
 
 <sub> </sub> <sub></sub>
 
2,0 đ
<b>3. </b>
(<i>4 đ</i>)


Cho tam giác <i>ABC</i> nhọn, khơng cân nội tiếp đường trịn

 

<i>O</i> . Gọi


<i>I</i> là tâm đường tròn nội tiếp tam giác<i>ABC</i>. Điểm <i>E</i> đối xứng với


<i>B</i> qua <i>IC</i>, <i>F</i> đối xứng với <i>C</i> qua <i>IB</i>. Gọi <i>M</i> là trung điểm của
cung <i>BAC</i> của đường tròn

 

<i>O</i> . Đường thẳng <i>MI</i> cắt đường tròn
ngoại tiếp tam giác <i>BIC</i> tại <i>N</i> (khác <i>I</i>). Đường thẳng <i>AI</i> cắt

 

<i>O</i>


tại điểm thứ hai <i>D</i> (khác<i>A</i>), đường thẳng qua <i>I</i> vng góc với


<i>AD</i> cắt <i>BC</i> tại <i>K</i>. Chứng minh rằng
a) <i>EF</i><i>OI</i>.



</div>
<span class='text_page_counter'>(13)</span><div class='page_container' data-page=13>

Sử dụng bổ đề: <i>Cho tam giác ABC nội tiếp đường tròn </i>

 

<i>O . Gọi I là </i>
<i>tâm đường tròn nội tiếp tam </i>


<i>giác đó. Gọi P là giao điểm </i>
<i> thứ hai, khác A của đường </i>
<i>thẳng AI và đường trịn </i>

 

<i>O . Khi đó P là tâm đường </i>
<i>tròn ngoại tiếp tam giác BIC</i>
<i>. </i>


a) Gọi <i>Q</i> là giao điểm thứ
hai khác <i>C</i> của đường
thẳng <i>CI</i> và đường tròn


 

<i>O</i>


Do <i>E</i> đối xứng với <i>B</i> qua <i>IC</i> và <i>IC</i> là phân giác trong của góc


<i>BCA</i>


 nên <i>E</i> nằm trên đường thẳng <i>AC</i> và <i>BEC</i> cân tại .<i>C</i>
<i>BOQ</i>


 cân tại <i>O</i> có 1


2


<i>BOQ</i> <i>BOA</i> <i>BCE</i>


     (do <i>ABC</i>nhọn)



Suy ra <i>QOB</i>đồng dạng với <i>ECB</i> <i>QO</i> <i>QB</i>


<i>BC</i> <i>EB</i>


   (1)


1,0 đ


Theo bổ đề ta có <i>QB</i><i>QA</i><i>QI</i> (2)


Do <i>F</i> đối xứng với <i>C</i> qua <i>IB</i> và <i>IB</i> là phân giác trong của góc


<i>ABC</i>


 nên <i>F</i> nằm trên đường thẳng <i>AB</i> và <i>BF</i><i>BC</i> (3)


Từ (1), (2), (3) ta có <i>QO</i> <i>QI</i>


<i>BF</i>  <i>EB</i>, lại có <i>QO</i><i>AB</i> và <i>QI</i> <i>BE</i>


(do tam giác <i>BCE</i>cân tại <i>C</i>và<i>QC</i>là phân giác trong của góc <i>BCE</i>


), suy ra <i>OQI</i>  <i>FBE</i> nên <i>BEF</i> đồng dạng <i>QIO</i>suy ra
.


<i>EF</i><i>OI</i>


1,0 đ



b) Khơng mất tính tổng qt, giả sử <i>AB</i><i>AC</i>, khi đó <i>K</i> nằm trên
tia đối của tia <i>BC</i>. Theo bổ đề <i>D</i> là tâm đường tròn ngoại tiếp


<i>BIC</i>


 , ta kí hiệu đường trịn này là

 

<i>D</i> . Có <i>KI</i> <i>AD</i> nên <i>KI</i> là
tiếp tuyến của đường tròn

 

<i>D</i> . Gọi <i>L</i> là giao điểm thứ hai (khác


<i>D</i>) của <i>KD</i> và đường trịn

 

<i>O</i> . Ta có 2


. .


<i>KI</i> <i>KB KC</i> <i>KL KD</i>.


</div>
<span class='text_page_counter'>(14)</span><div class='page_container' data-page=14>

Suy ra <i>KIL</i>đồng dạng với <sub></sub><i><sub>KDI</sub></i><sub> </sub><i><sub>KLI</sub></i><sub> </sub><i><sub>KID</sub></i><sub></sub><sub>90</sub>0<sub> (4) </sub>
<i>I</i> là tâm đường tròn nội tiếp <i>ABC</i> nên <i>AI</i> là phân giác trong của
góc <i>BAC</i>. Do đó <i>D</i> là trung điểm cung <i>BC</i> không chứa điểm <i>A</i>


của ( )<i>O</i> , kết hợp <i>M</i> là trung điểm cung <i>BAC</i> của ( )<i>O</i>


suy ra <i>DM</i> là đường kính của đường trịn ( )<i>O</i> nên ta có


0


90


<i>MLK</i>


  (5).



Từ (4), (5) suy ra ba điểm <i>M I L</i>, , thẳng hàng. Do <i>N</i> và <i>I</i> đều thuộc
( )<i>D</i> nên <i>DN</i><i>DI</i>, vì vậy <i>N</i> và <i>I</i> đối xứng nhau qua đường thẳng


<i>LD</i>. Mà <i><sub>K</sub></i><sub></sub><i><sub>LD</sub></i><sub> </sub><i><sub>KND</sub></i><sub> </sub><i><sub>KID</sub></i><sub></sub><sub>90</sub>0<sub> hay </sub><i><sub>KN</sub></i><sub> là tiếp tuyến của </sub>


đường tròn ( ).<i>D</i>


1,0 đ


<b>4. </b>
(<i>4 đ</i>)


Cho các số nguyên dương <i>a a</i><sub>1</sub>, <sub>2</sub>, ...,<i>a</i><sub>2018</sub>. Xét tập


 

 

 





  1   2    2018   


2018 1 1 1 2 ... 1 2018; 1,...,2018 .


<i>k</i> <i>k</i> <i>k</i>


<i>i</i>


<i>S</i> <i>a</i> <i>a</i> <i>a</i> <i>k</i> <i>i</i>


Chứng minh rằng tích của tất cả các phần tử thuộc <i>S</i><sub>2018</sub>là một số
chính phương.



Với mỗi <i>n</i>

1, 2, 3,..., 2018

, ta kí hiệu


 

 

 



1 2



1 2


1 <i>k</i> 1 <i>k</i> ... 1 <i>kn</i> ; , 1,..., .


<i>n</i> <i>n</i> <i>i</i>


<i>S</i> <sub></sub> <sub></sub> <i>a</i> <sub> </sub> <i>a</i> <sub></sub> <sub> </sub> <i>a</i> <i>k</i> <sub></sub><sub></sub> <sub> </sub><i>i</i> <i>n</i>


Nhận xét rằng:


+<i>S<sub>n</sub></i>có 2<i>n</i> phần tử là các số thực đối nhau.
+ Với mỗi <i>e</i><i>S<sub>n</sub></i><sub>1</sub> thì <i>e</i> <i>an</i> <i>Sn</i>.


Đặt


<i>n</i>


<i>n</i>
<i>e S</i>


<i>A</i> <i>e</i>





<sub></sub>

là tích của tất cả các phần tử thuộc <i>S<sub>n</sub></i>. Ta chứng
minh <i>A<sub>n</sub></i> là số chính phương với mọi <i>n</i>

1, 2, 3,..., 2018

, từ đó ta
cũng có điều phải chứng minh.


</div>
<span class='text_page_counter'>(15)</span><div class='page_container' data-page=15>

Xét đa thức

 



<i>n</i>


<i>n</i>


<i>e S</i>


<i>P x</i> <i>x</i> <i>e</i>




<sub></sub>

 thì <i>A<sub>n</sub></i> <i>P<sub>n</sub></i>

 

0 .
Theo nhận xét trên ta có


 





1


1 1 .


<i>n</i>


<i>n</i> <i>n</i> <i>n</i> <i>n</i> <i>n</i> <i>n</i> <i>n</i>



<i>e S</i>


<i>P x</i> <i>x e</i> <i>a</i> <i>x</i> <i>e</i> <i>a</i> <i>P</i> <i>x</i> <i>a</i> <i>P</i> <i>x</i> <i>a</i>




 




<sub></sub>

      




1 1; 1


<i>S</i>   <i>a</i> <i>a</i> ta có

<sub> </sub>



2


1 1 1 1 <i>x</i> .


<i>P x</i> <i>x</i> <i>a</i> <i>x</i> <i>a</i> <i>x</i> <i>a</i> <sub> </sub>


 


     




2 1 2; 1 2; 1 2; 1 2



<i>S</i>   <i>a</i>  <i>a</i>  <i>a</i>  <i>a</i> <i>a</i>  <i>a</i> <i>a</i>  <i>a</i> ta có


 











2 1 2 1 2 1 2 1 2


2


4 2


1 2 1 2


2 <i><sub>x</sub></i> .


<i>P x</i> <i>x</i> <i>a</i> <i>a</i> <i>x</i> <i>a</i> <i>a</i> <i>x</i> <i>a</i> <i>a</i> <i>x</i> <i>a</i> <i>a</i>


<i>x</i> <i>a</i> <i>a x</i> <i>a</i> <i>a</i> <sub> </sub>


 


        


     


Mà <i>P x</i><sub>2</sub>

 <i>a</i><sub>3</sub>

<i>A x</i><sub>2</sub>

 

 <i>a B x</i><sub>3</sub>. <sub>2</sub>

 

; <i>P x</i><sub>2</sub>

 <i>a</i><sub>3</sub>

<i>A x</i><sub>2</sub>

 

 <i>a B x</i><sub>3</sub>. <sub>2</sub>

 


với <i>A x B x</i><sub>2</sub>

<sub> </sub>

; <sub>2</sub>

<sub> </sub>

<sub> </sub><i><sub>x</sub></i>


 



 do <i>P x</i><sub>2</sub>

<sub> </sub>

<sub> </sub><i><sub>x</sub></i> .


 


 Từ đó ta được


 

 

2

 

2

 



3 2 3 2 3 2 3. 2 <i>x</i> .


<i>P x</i> <i>P x</i> <i>a</i> <i>P x</i> <i>a</i> <i>A</i> <i>x</i> <i>a B</i> <i>x</i> <sub> </sub>


 


     


Bằng phương pháp quy nạp ta chứng minh được


 

1, 2, 3,..., 2018 .



<i>n</i> <i>x</i>


<i>P x</i> <sub> </sub> <i>n</i>
 


  


2,0 đ



Lại có <i>S<sub>n</sub></i>chỉ chứa các số đối nhau, giả sử là <sub>1</sub> <sub>2</sub> 1


2


; ; ...; <i>n</i> .


<i>e</i> <i>e</i> <i>e</i> 


  


 





1 1


2 2


2 2


1 1


<i>n</i> <i>n</i>


<i>n</i>


<i>n</i> <i>i</i> <i>i</i> <i>i</i>


<i>e S</i> <i>i</i> <i>i</i>


<i>P x</i> <i>x</i> <i>e</i> <i>x e</i> <i>x e</i> <i>x</i> <i>e</i>



 


  


<sub></sub>

 

<sub></sub>

  

<sub></sub>



suy ra <i>P x<sub>n</sub></i>

 

<i>P<sub>n</sub></i>

 

<i>x</i> <i>x</i>. Do đó

 

 

2


<i>n</i> <i>n</i>


<i>P x</i> <i>Q x</i> với <i>Q x<sub>n</sub></i>

 

<sub> </sub><i><sub>x</sub></i> .


 



 

 



 0  <sub>1</sub>  <sub>1</sub>  2<sub>1</sub>  2 <sub>1</sub>


<i>n</i> <i>n</i> <i>n</i> <i>n</i> <i>n</i> <i>n</i> <i>n</i> <i>n</i> <i>n</i> <i>n</i>


<i>A</i> <i>P</i> <i>P</i> <i>a</i> <i>P</i> <i>a</i> <i>P</i> <i>a</i> <i>Q</i> <i>a</i>


là số chính phương.


1,0 đ


<b>5. </b>
(<i>4 đ</i>)



Cho đa giác lồi <i>P</i>. Bạn An muốn ghi vào mỗi đỉnh của <i>P</i> một số
nguyên dương sao cho các điều kiện sau được đồng thời thỏa mãn:
i) Trong các số được ghi, có ít nhất một số chẵn;


ii) Tổng của ba số được ghi ở ba đỉnh liên tiếp tùy ý là một số lẻ.
Chứng minh rằng bạn An có thể thực hiện được cách ghi như
trên khi và chỉ khi số đỉnh của <i>P</i> chia hết cho 3.


<i>Điều kiện cần: </i>Giả sử bạn An đã thực hiện được cách ghi thỏa
mãn đề bài. Xuất phát từ một đỉnh nào đó, lần lượt, theo chiều
kim đồng hồ, kí hiệu các đỉnh của <i>P</i> là <i>A A</i><sub>1</sub>, <sub>2</sub>,...,<i>A<sub>n</sub></i>. Với mỗi


1, 2,...,



<i>i</i> <i>n</i> , kí hiệu <i>a<sub>i</sub></i>là số được bạn An ghi vào đỉnh <i>A<sub>i</sub></i>.
Theo i), trong các số <i>a a</i><sub>1</sub>, <sub>2</sub>,...,<i>a<sub>n</sub></i>có ít nhất một số chẵn. Không mất


tính tổng quát, có thể giả sử <i>a</i><sub>1</sub>chẵn.


</div>
<span class='text_page_counter'>(16)</span><div class='page_container' data-page=16>

Trong phần trình bày sau đây ta coi các chỉ số <i>n</i><i>k</i>, 1<i>k</i><i>n</i>, và
chỉ số <i>k</i>là một (các chỉ số được xét theo modulo<i>n</i>)


<i>Nhận xét: </i>Với mọi <i>i j</i>, 

1; 2; 3;...;<i>n</i>

, nếu

<i>j i</i>

chia hết cho 3 thì
<i>i</i>


<i>a</i> và <i>a<sub>j</sub></i> có cùng tính chẵn lẻ. Thật vậy, với <i>i</i>

1; 2; 3;...;<i>n</i>

theo
ii) ta có <i>a<sub>i</sub></i> <i>a<sub>i</sub></i><sub>1</sub><i>a<sub>i</sub></i><sub>2</sub> và <i>ai</i>1<i>ai</i>2 <i>ai</i>3là các số lẻ nên


<i>ai</i>1<i>ai</i>2<i>ai</i>3

 

 <i>ai</i><i>ai</i>1<i>ai</i>2

<i>ai</i>3<i>ai</i> là một số chẵn. Vì thế
<i>i</i>


<i>a</i> và <i>a<sub>i</sub></i><sub></sub><sub>3</sub>có cùng tính chẵn lẻ. Từ đó, do<i>i</i> tùy ý nên ta có nhận xét
trên


Xét các trường hợp sau:


+ Nếu <i>n</i> 3<i>k</i> 1,<i>k</i> .


   Trong trường hợp này vì <i>a</i><sub>1</sub>chẵn nên theo


nhận xét thì <i>a</i><sub>3</sub><i><sub>k</sub></i><sub>1</sub> cũng là số chẵn, lại có <i>a</i>3<i>k</i>1<i>a</i>1<i>a</i>2 là số lẻ, suy


ra <i>a</i><sub>2</sub> là số lẻ. Do đó theo nhận xét <i>a</i><sub>3</sub><i><sub>k</sub></i><sub>2</sub> <i>a</i><sub>1</sub>là số lẻ, ta nhận được


mâu thuẫn, chứng tỏ <i>n</i>3<i>k</i>1.


+ Nếu <i>n</i> 3<i>k</i> 2,<i>k</i> .


   Trong trường hợp này vì <i>a</i><sub>1</sub>chẵn nên theo


nhận xét thì <i>a</i><sub>3</sub><i><sub>k</sub></i><sub>1</sub> cũng là số chẵn, lại có <i>a</i>3<i>k</i>1<i>a</i>3<i>k</i>2<i>a</i>1 là số lẻ,


suy ra <i>a</i><sub>3</sub><i><sub>k</sub></i><sub>2</sub> là số lẻ. Do đó theo nhận xét, <i>a</i>2là số lẻ, suy ra
3<i>k</i> 2 1 2


<i>a</i>  <i>a</i> <i>a</i> là số chẵn, trái với ii), ta nhận được mâu thuẫn,


chứng tỏ <i>n</i>3<i>k</i>2.


Vì các trường hợp trên khơng xảy ra nên phải có <i>n</i> chia hết cho


3.


1,0 đ


<i> Điều kiện đủ</i>: Giả sử <i>P</i>có <i>n</i>3<i>k</i> đỉnh

<i>k</i> 

.


 Xuất phát từ một


đỉnh nào đó, lần lượt, theo chiều kim đồng hồ, kí hiệu các đỉnh
của <i>P</i> là <i>A A</i><sub>1</sub>, <sub>2</sub>,...,<i>A</i><sub>3</sub><i><sub>k</sub></i>.


Xét cách ghi số như sau của bạn An: Với mỗi <i>i</i>

1; 2; 3; ...; 3<i>k</i>

,
ghi vào đỉnh <i>A<sub>i</sub></i> một số nguyên dương lẻ nếu <i>i</i>2 (mod 3) và ghi


vào đỉnh <i>A<sub>i</sub></i> một số nguyên dương chẵn trong trường hợp còn


lại.


Rõ ràng cách ghi như trên thỏa mãn điều kiện i) của đề bài.


1,0 đ


Do <i>n</i>0 (mod 3) nên trong ba đỉnh liên tiếp tùy ý của <i>P</i> ln có
một đỉnh có chỉ số chia hết cho 3 dư 1, một đỉnh có chỉ số chia cho 3
dư 2 và một đỉnh có chỉ số chia hết cho 3. Vì thế, với cách ghi số
nêu trên ta ln có tổng của ba số được ghi ở 3 đỉnh liên tiếp của


<i>P</i> là một số lẻ (do bằng tổng của một số lẻ và hai số chẵn). Vì vậy
cách ghi số như trên thỏa mãn điều kiện ii) của đề bài.



</div>
<span class='text_page_counter'>(17)</span><div class='page_container' data-page=17>

SỞ GIÁO DỤC VAØ ĐAØO TẠO


BẮC GIANG



<b>ĐỀ THI CHÍNH THỨC </b>


KỲ THI CHỌN ĐỘI TUYỂN DỰ THI


HỌC SINH GIỎI QUỐC GIA NĂM 2018–2019



<b>Mơn: TỐN</b>


Thời gian:<b> 180 </b>phút (khơng kể thời gian giao đề)
Ngày thi: 15/9/2018


<b>Câu 1 </b><i>(4 đim)</i><b>. Giải hệ phương trình </b>


2

2



1 2


2


1 1 3 1 .


<i>y</i> <i>x</i>


<i>x</i> <i>y</i>


<i>x</i>


<i>y</i> <i>x</i> <i>x</i>





  






 <sub></sub> <sub></sub> <sub></sub> <sub></sub>





<b>Câu 2 </b><i>(4 điểm)</i><b>. Cho hai dãy số </b>

 

<i>x<sub>n</sub></i> và

 

<i>y<sub>n</sub></i> thỏa mãn


1


1 2018; 1 201 1 1


2


9, <i>n</i> <i>n</i> ; , 2.


<i>n</i> <i>n</i> <i>n n</i>


<i>x</i> <i>y</i>


<i>x</i>



<i>x</i> <i>y</i>   <i>y</i> <i>x y</i> <i>n</i>






   


 


Chứng minh rằng các dãy số

 

<i>x<sub>n</sub></i> và

 

<i>y<sub>n</sub></i> đều có giới hạn hữu hạn và tìm các
giới hạn đó.


<b>Câu 3 </b><i>(6 điểm)</i> Cho tam giác <i>ABC</i> nhọn, khơng cân, ngoại tiếp đường trịn (<i>I</i>). Gọi


<i>D, E, F</i> lần lượt là các tiếp điểm của (I) với <i>BC</i>, <i>CA</i>, <i>AB</i>.


a) Gọi <i>S</i> là giao điểm của hai đường thẳng <i>EF</i> và <i>BC</i>. Chứng minh rằng <i>SI</i><i>AD</i>.


b) Gọi <i>X</i> là giao điểm khác <i>A</i> của hai đường tròn (<i>ABE</i>) và (<i>ACF</i>), <i>Y</i> là giao điểm
khác <i>B</i> của hai đường tròn (<i>BAD</i>) và (<i>BCF</i>), <i>Z</i> là giao điểm khác <i>C</i> của hai
đường tròn (<i>CAD</i>) và (<i>CBE</i>). Chứng minh rằng <i>AX</i>, <i>BY</i>, <i>CZ</i> đồng quy.
<b>Câu 4 </b><i>(3 điểm)</i><b>. Với mỗi tập </b><i>A</i> gồm <i>n</i> điểm phân biệt trong mặt phẳng

<i>n</i>2

, kí


hiệu <i>T A</i>

 

là tập hợp các vectơ có điểm đầu và điểm cuối đều thuộc .<i>A</i> Hãy
xác định giá trị lớn nhất và giá trị nhỏ nhất của <i>T A</i>

 

. (Kí hiệu <i>T A</i>

 

là số
phần tử của tập hợp <i>T A</i>

 

).


<b>Câu 5 </b><i>(3 điểm)</i><b>. Cho </b><i>p</i> là số nguyên tố lẻ, <i>k</i> là số nguyên dương.
Đặt <i>S</i>(<i>p</i>, <i>k</i>) =



1


1


<i>p</i>
<i>k</i>
<i>i</i>


<i>i</i>






. Chứng minh rằng


a) <i>S</i>(<i>p,k</i>)  -1 (mod <i>p</i>) <i>k</i> chia hết cho <i>p</i> – 1.


</div>
<span class='text_page_counter'>(18)</span><div class='page_container' data-page=18>

<b>ĐÁP ÁN – THANG ĐIỂM </b>



<b>Câu 1 </b><i>(4 điểm)</i><b> Giải phương trình </b>


2

2



1 2


2


1 1 3 1



<i>y</i> <i>x</i>


<i>x</i> <i>y</i>


<i>x</i>


<i>y</i> <i>x</i> <i>x</i>




  






 <sub></sub> <sub></sub> <sub></sub> <sub></sub>





<b>Nội dung </b> <b>Điểm </b>


Phương trình đầu tương đương <i>y</i>2<i>y</i>

<i>x</i>2<i>x</i>

2<i>x x</i> 0.
Coi đây là phương trình bậc hai ẩn y, tham số x, ta có:


1


2



.
2


<i>y</i> <i>x</i>


<i>y</i> <i>x</i>


 <sub> </sub>






1,0


Với <i>y</i><sub>1</sub>  <i>x</i>, thay vào phương trình thứ hai, vơ nghiệm


Với <i>y</i><sub>1</sub>2<i>x</i>, thay vào phương trình thứ hai được 2 1 2 (*)


2 3


<i>x</i>
<i>x</i>


<i>x</i>


  





1,0


Xét hàm số <i>f x</i>

 

 <i>x</i>21đồng biến trên

0;

,

 

2


2 3


<i>x</i>
<i>g x</i>


<i>x</i>


 nghịch


biến trên

0;

nên (*) có khơng q một nghiệm.


1,0


Nhẩm nghiệm thấy (*) có nghiệm <i>x</i> 3


Hệ có nghiệm

<i>x y</i>,

3; 2 3



1,0


<b>Câu 2 </b><i>(4 điểm)</i><b> Cho hai dãy số </b>

 

<i>x<sub>n</sub></i> và

 

<i>y<sub>n</sub></i> thỏa mãn


1
1



1


1 1


2018; 201


2


9, <i>n</i> <i>n</i> ; , 2.


<i>n</i> <i>n</i> <i>n n</i>


<i>x</i> <i>y</i>


<i>x</i>


<i>x</i> <i>y</i>   <i>y</i> <i>x y</i> <i>n</i>






   


 


Chứng minh rằng các dãy số

 

<i>x<sub>n</sub></i> và

 

<i>y<sub>n</sub></i> đều có giới hạn hữu hạn và tìm các
giới hạn đó.


<b>Nội dung </b> <b>Điểm </b>



Đặt 1


1 1
1


2018


cos , 0; cos .


2019 2


<i>x</i>


<i>a a</i> <i>x</i> <i>y</i> <i>a</i>


<i>y</i>




 


  <sub></sub> <sub></sub> 


 


Ta tính được





  




2 2


2 2 3 2


2 2


2019 cos ; 2019 cos , 2019 cos cos ;


2 2 2 2


2019 cos cos


2 2


<i>a</i> <i>a</i> <i>a</i> <i>a</i>


<i>x</i> <i>y</i> <i>x</i>


<i>a</i> <i>a</i>


<i>y</i>


</div>
<span class='text_page_counter'>(19)</span><div class='page_container' data-page=19>

Quy nạp ta được


 








2


2 2 1


2 2 1


2019 cos cos ...cos cos ;


2 2 2 2


2019 cos cos ...cos


2 2 2


<i>n</i> <i>n</i> <i>n</i>


<i>n</i>


<i>a</i> <i>a</i> <i>a</i> <i>a</i>


<i>x</i>


<i>a</i> <i>a</i> <i>a</i>


<i>y</i>



Do sin 2 2 sin cos  nên


1 1


1


1


2019.sin 2019. sin


cot ;


2 2 <sub>2</sub> <sub>sin</sub>


2


<i>n</i> <i>n</i> <i>n</i> <i>n</i>


<i>n</i>


<i>n</i>


<i>a</i> <i>a</i> <i>a</i>


<i>x</i> <i>y</i>


<i>a</i>


 







  .


Từ đó suy ra


2019 sin 4037


lim lim


2018
arccos


2019


<i>n</i> <i>n</i>


<i>a</i>


<i>x</i> <i>y</i>


<i>a</i>


  


1,0



1,0


<b>Câu 3 </b><i>(6 điểm)</i> Cho tam giác <i>ABC</i> nhọn, khơng cân, ngoại tiếp đường trịn (<i>I</i>). Gọi


<i>D, E, F</i> lần lượt là các tiếp điểm của (I) với <i>BC</i>, <i>CA</i>, <i>AB</i>.


a) Gọi <i>S</i> là giao điểm của hai đường thẳng <i>EF</i> và <i>BC</i>. Chứng minh rằng <i>SI</i><i>AD</i>.


b) Gọi <i>X</i> là giao điểm khác <i>A</i> của hai đường tròn (<i>ABE</i>) và (<i>ACF</i>), <i>Y</i> là giao điểm
khác <i>B</i> của hai đường tròn (<i>BAD</i>) và (<i>BCF</i>), <i>Z</i> là giao điểm khác <i>C</i> của hai
đường tròn (<i>CAD</i>) và (<i>CBE</i>). Chứng minh rằng <i>AX</i>, <i>BY</i>, <i>CZ</i> đồng quy.


<b>Nội dung </b> <b>Điểm </b>


a) Gọi G là giao điểm của AD và (I).
Khi đó tứ giác GEDF điều hồ
Do đó tiếp tuyến tại D, G của (I) và
FE đồng quy tại S


Từ đó có điều phải chứng minh.


1,5


b) Ta có:


sin sin


sin sin


<i>XAB</i> <i>XEB</i> <i>BX</i>



<i>XAC</i> <i>XBE</i> <i>XE</i>


 


 


 


Mặt khác, X là điểm Miquel của tứ giác toàn phần BAECPF


<i>XBF</i> =  1800<sub> – </sub><sub></sub><i><sub>XPF</sub></i><sub> = </sub><sub></sub><i><sub>XCE</sub></i><sub>, </sub><sub></sub><i><sub>FXB</sub></i><sub> = </sub><sub></sub><i><sub>FPB</sub></i><sub> = </sub><sub></sub><i><sub>CPE </sub></i>


 <i>FXB</i><i>CXE</i>


<i>XB</i> <i>BF</i>


<i>XE</i><i>CE</i>


sin
sin


<i>XAB</i> <i>BF</i>
<i>XAC</i> <i>CE</i>





Từ đó dễ dàng suy ra đpcm.



</div>
<span class='text_page_counter'>(20)</span><div class='page_container' data-page=20>

<b>Câu 4. </b><i>(4 điểm)</i> Với mỗi tập <i>A</i> gồm <i>n</i> điểm phân biệt trong mặt phẳng

<i>n</i>2

, kí
hiệu <i>T A</i>

 

là tập hợp các vectơ có điểm đầu và điểm cuối đều thuộc .<i>A</i> Hãy
xác định giá trị lớn nhất và giá trị nhỏ nhất của <i>T A</i>

 

. (Kí hiệu <i>T A</i>

 

là số
phần tử của tập hợp <i>T A</i>

 

).


<b>Nội dung </b> <b>Điểm </b>


Số đoạn thẳng với hai đầu mút khác nhau thuộc <i>A</i> là

1



2


<i>n n</i>


nên số
vectơ khác 0




thuộc <i>A</i> không vượt quá <i>n n</i>

1

. Do đó kể cả 0




thì


 

2 <sub>1.</sub>


<i>T A</i> <i>n</i> <i>n</i>


Ta chứng minh tồn tại cấu hình sao cho <i>T A</i>

 

<i>n</i>2 <i>n</i>1.


Xét n điểm A1, A2, …, An sao cho A1, A2, A3, …, An d và thoả mãn điều


kiện A1Aj < AjAj+1 với j = 2, 3, 4, …, n.


Khi đó dễ dàng cm được <i><sub>T A</sub></i>

 

<sub></sub><i><sub>n</sub></i>2 <sub></sub><i><sub>n</sub></i><sub></sub><sub>1.</sub>


1,0


Xét tập <i>A</i> gồm <i>n</i> điểm <i>A A</i><sub>1</sub>, <sub>2</sub>,...,<i>A<sub>n</sub></i>. Giả sử <sub>1</sub>



1max


<i>n</i> <i><sub>i j n</sub></i> <i>i</i> <i>j</i>


<i>A A</i> <i>A A</i>


  




Gọi d là đường thẳng qua <i>A</i><sub>1</sub> và vng góc
với <i>A A</i><sub>1</sub> <i><sub>n</sub></i>.


Khi đó tất cả các điểm thuộc tập <i>A</i>đều thuộc
nửa mặt phẳng chứa <i>A<sub>n</sub></i>có bờ là d, và chỉ có


1


<i>A</i> thuộc d.



0,5


Như vậy sẽ có <i>2(n-1)</i> vectơ khác 0




và đôi một phân biệt là


1 2,..., 1 <i>n</i>, 2 1,..., <i>n</i> 1


<i>A A</i> <i>A A A A</i> <i>A A</i>


   

 

2

1

1 2 1.


<i>T A</i>  <i>n</i>   <i>n</i>


0,5


<b>. </b>



1
<i>A</i>


<i>n</i>


</div>
<span class='text_page_counter'>(21)</span><div class='page_container' data-page=21>

Chỉ cần chỉ ra một tập hợp mà <i>T A</i>

 

2<i>n</i>1.


Trên trục số đặt <i>A<sub>k</sub></i>vào điểm có tọa độ <i>k</i> thì với mọi <i>A A</i><i><sub>i</sub></i> <i><sub>j</sub></i> đều tồn tại <i>k</i>



sao cho <i>A A</i><sub>1</sub> <i><sub>k</sub></i>  <i>A A</i><sub>1</sub> <i><sub>k</sub></i>
 


hoặc <i>A A<sub>k</sub></i> <sub>1</sub> <i>A A</i><sub>1</sub> <i><sub>k</sub></i>
 


, nên min<i>T A</i>

 

2<i>n</i>1.


1,0


<b>Câu 5 </b><i>(3 điểm)</i> Cho <i>p</i> là số nguyên tố lẻ, <i>k</i> là số nguyên dương.
Đặt <i>S</i>(<i>p</i>, <i>k</i>) =


1


1


<i>p</i>
<i>k</i>
<i>i</i>


<i>i</i>






. Chứng minh rằng



a) <i>S</i>(<i>p,k</i>)  -1 (mod <i>p</i>) <i>k</i> chia hết cho <i>p</i> – 1.


b) <i>S</i>(<i>p,k</i>)  0 (mod <i>p</i>) <i>k</i> không chia hết cho <i>p</i> – 1.


<b>Nội dung </b> <b>Điểm </b>


a) Nếu k  p - 1 thì


1 1


( ) 1(mod ), 1, 1.


<i>k</i>


<i>p</i> <i>p</i>


<i>k</i>


<i>i</i>  <i>p</i>    <i>p</i>  <i>i</i> <i>p</i>


Do đó


1


1


( , ) 1 1 1(mod ).


<i>p</i>
<i>i</i>



<i>S p k</i> <i>p</i> <i>p</i>






<sub></sub>

   


b) Nếu k ≠ p – 1 thì k = (p – 1)q + r, 1 ≤ r ≤ p – 2, q  .


(mod ) ( , ) ( , )(mod ).


<i>k</i> <i>r</i>


<i>i</i> <i>i</i> <i>p</i> <i>S p k</i> <i>S p r</i> <i>p</i>


   


Ta có phương trình xr<sub></sub><sub> 1 (mod p) có khơng q r nghiệm. </sub>


Mà r  {1, 2, …, p – 2} nên tồn tại a  {1, 2, …, p – 1} sao cho


1(mod ).


<i>r</i>


<i>a</i>  <i>p</i>


Vì (a, p) = 1 nên { }<i>ia<sub>i</sub>p</i><sub></sub><sub>1</sub>1 là hệ thặng dư thu gọn (mod p).


Do đó


1 1 1


1 1 1


( , ) ( ) (mod )


( 1) ( , ) 0(mod ).


<i>p</i> <i>p</i> <i>p</i>


<i>r</i> <i>r</i> <i>r</i> <i>r</i>


<i>i</i> <i>i</i> <i>i</i>


<i>r</i>


<i>S p r</i> <i>i</i> <i>ia</i> <i>a</i> <i>i</i> <i>p</i>


<i>a</i> <i>S p r</i> <i>p</i>


  


  


  


  





Do (ar<sub> – 1, p) = 1 </sub><sub></sub><sub> S(p, r) </sub>


 0 (mod p).


0.5


1,0


0.75


</div>
<span class='text_page_counter'>(22)</span><div class='page_container' data-page=22>

<b>UBND TỈNH BẮC NINH </b>



SỞ GIÁO DỤC VÀ ĐÀO TẠO



<b>ĐỀ THI CHÍNH THỨC </b>


ĐỀ THI CHỌN ĐỘI TUYỂN DỰ THI


HỌC SINH GIỎI QUỐC GIA NĂM 2019



<b>Mơn: TỐN</b>


Thời gian:<b> 180 </b>phút (không kể thời gian giao đề)
Ngày thi: 04/10/2018 (Ngày thi thứ nhất)


<b>Câu 1. </b><i>(4,0 điểm)</i>Giải hệ phương trình sau trên tập số thực:


2 2



2 2


2 5 2 2 5


2
2


2 2 .


log ( 2 11) log ( 2 12)


<i>xy</i> <i>x</i> <i>y</i>


<i>xy</i> <i>x</i> <i>y</i>


<i>x</i> <i>y</i>


<i>x</i> <i>y</i> <i>y</i> <i>x</i>


 


 <sub></sub> <sub></sub> <sub></sub>


  


 <sub></sub>


 <sub></sub> <sub></sub> <sub></sub> <sub></sub> <sub></sub>






<b>Câu 2. </b><i>(4,0 điểm)</i>


Cho dãy số thực ( )<i>a<sub>n</sub></i> xác định bởi *


1 1


1 2


1


1, 2 , .


...


<i>n</i>


<i>n</i>


<i>a</i> <i>a</i> <i>n</i>


<i>a</i> <i>a</i> <i>a</i>




    


   <b></b>



Chứng minh rằng dãy ( )<i>b<sub>n</sub></i> xác định bởi *


1 2 ... ,


<i>n</i> <i>n</i>


<i>b</i> <i>a</i> <i>a</i>  <i>a</i>  <i>n</i> <b></b> có giới hạn


hữu hạn và tìm giới hạn đó.


<b>Câu 3. </b><i><b>(6,0 điểm)</b></i> Cho tam giác<i>ABC</i>nhọn, khơng cân, nội tiếp đường trịn ( ).<i>O</i> Gọi


<i>H</i> là trực tâm của tam giác <i>ABC</i>.


a) Gọi ,<i>I K</i> lần lượt là trung điểm của <i>AB AC</i>, ; <i>B C</i><sub>1</sub>, <sub>1</sub> lần lượt là chân đường cao
kẻ từ ,<i>B C</i> của tam giác <i>ABC</i>. Đường thẳng <i>IK</i> cắt <i>B C</i><sub>1 1</sub> tại ,<i>U</i> đường thẳng


<i>OH</i> cắt <i>IK</i> tại .<i>V</i> Chứng minh rằng <i>V</i> là trực tâm tam giác <i>AHU</i>.


b) Gọi <i>M</i> là trung điểm của đoạn thẳng <i>AH</i>, <i>T</i> là giao điểm của tiếp tuyến tại


<i>A</i> của đường tròn ( )<i>O</i> với đường thẳng <i>BC</i>, <i>P</i> là hình chiếu vng góc của


<i>O</i> trên đường thẳng <i>TM</i>. Chứng minh rằng trung điểm của đoạn thẳng <i>MP</i>


nằm trên đường tròn <i>Euler</i> của tam giác <i>ABC</i>.


 (<i>Chú ý: </i>Đường tròn <i>Euler</i> của tam giác <i>ABC</i>là đường tròn qua 9 điểm gồm trung
điểm các cạnh, chân đường cao và trung điểm của các đoạn thẳng nối các đỉnh với


trực tâm tam giác <i>ABC</i>).


<b>Câu 4. </b><i>(6,0 điểm)</i>


a) Tìm tất cả các hàm :<i>f</i> <b></b><b></b> thỏa mãn


2 2


( ( ) ) ( ), ,


<i>f xf y</i> <i>x</i> <i>xy</i> <i>f x</i> <i>x y</i><b></b>.


</div>
<span class='text_page_counter'>(23)</span><div class='page_container' data-page=23>

<b>ĐÁP ÁN – THANG ĐIỂM</b>



<b>Câu </b> <b>Đáp án </b> <b>Điểm </b>


<b>1. </b>


Giải hệ phương trình sau trên tập số thực:


 


 


 
 <sub></sub> <sub></sub> <sub></sub>
  
 <sub></sub>

 <sub></sub> <sub></sub> <sub></sub> <sub></sub> <sub></sub>



2 2
2 2


2 5 <sub>2 2</sub> <sub>5</sub>


2
2


1


2 2


log ( 2 11) log ( 2 12) 2


<i>xy</i> <i>x</i> <i>y</i>


<i>xy</i> <i>x</i> <i>y</i>


<i>x</i> <i>y</i>


<i>x</i> <i>y</i> <i>y</i> <i>x</i>


<b>4,0 </b>
ĐK:
2 2
2
0
2 2


<i>xy</i> <i>x</i> <i>y</i> <i>x</i> <i>y</i>



<i>xy</i>
<i>x</i> <i>y</i>
 
   

2 2
2 2
( ) ( )
0
2( )
2
2


<i>x</i> <i>y</i> <i>x</i> <i>y</i>


<i>x</i> <i>y</i> <i><sub>x</sub></i> <i><sub>y</sub></i>


<i>xy</i>
  
  
  <sub></sub> 
  
 


  1,0


Từ (*) và ĐK, ta có:






Do đó


Do vậy phương trình (1) tương đương


1,0


Thay vào phương trình (2) ta có



Đặt
Đặt
1,0
2 2

0;

0



2

11

0;

2

12

0



<i>xy</i>

<i>x</i>

<i>y</i>



<i>x</i>

<i>x</i>

<i>y</i>

<i>x</i>



 

 





 





2 2
(*)
2

<i>x</i> <i>y</i>
<i>x</i> <i>y</i>


<i>x</i> <i>y</i> <i>xy</i>


 


 <sub></sub>
   



0



<i>x</i>

 

<i>y</i>



2 2 2 2


(*) 2. (*).


2 2


<i>x</i> <i>y</i> <i>x</i> <i>y</i>


<i>VP</i>    <i>xy</i>  <sub></sub>  <i>xy</i><sub></sub><sub></sub>  <i>x</i> <i>y</i> <i>VT</i>


 


(*) 2. (*).



<i>VP</i>   <i>xy</i>   <i>xy</i><sub></sub>  <i>x</i> <i>y</i> <i>VT</i>


 


(*)  <i>x</i> <i>y</i> 0


0.



<i>x</i>

 

<i>y</i>



2 2


2 5 2 2 5


log<sub></sub> (<i>x</i> 2<i>x</i> 11) log (<i>x</i> 2<i>x</i> 12)




    


2


2


2 2 5 (2 5) 1 <i>a</i> 1


 <sub></sub>


  <sub></sub>     



 <sub></sub>


 


2 2


2 5 2 2 5


log (<i>x</i> 2<i>x</i> 11) log (<i>x</i> 2<i>x</i> 12) 2<i>t</i>


</div>
<span class='text_page_counter'>(24)</span><div class='page_container' data-page=24>



Do đó


(t/m)


Vậy hệ phương trình có nghiệm

<i>x y</i>;



2 5; 2 5 , 2

 

2 5; 22 5



1,0


<b>2. </b>


Cho dãy số thực ( )<i>a<sub>n</sub></i> xác định bởi


*


1 1



1 2


1


1, 2 , .


...


<i>n</i>


<i>n</i>


<i>a</i> <i>a</i> <i>n</i>


<i>a</i> <i>a</i> <i>a</i>




    


   <b></b>


Chứng minh rằng dãy ( )<i>b<sub>n</sub></i> xác định bởi <i>b<sub>n</sub></i> <i>a</i><sub>1</sub><i>a</i><sub>2</sub>...<i>a<sub>n</sub></i>, <i>n</i> <b></b>*


có giới hạn hữu hạn và tìm giới hạn đó.


<b>4,0 </b>


Ta có <i><sub>n</sub></i> <sub>1</sub> <i><sub>n</sub></i> <i><sub>n</sub></i> <sub>1</sub> <i><sub>n</sub></i> 1 2



<i>n</i>


<i>b</i> <i>b</i> <i>a</i> <i>b</i>


<i>b</i>


       với <i>b</i>1 1.


Suy ra <i>b<sub>n</sub></i>2 2 0, với mọi <i>n</i>*


và <sub>2</sub> 2 2 , <sub>3</sub> 3 1 2 1.


2


<i>b</i>   <i>b</i>  <sub></sub>  <sub></sub>


 


1,0


Xét hàm <i>f x</i>

 

<i>x</i> 1 2


<i>x</i>


   trên

0; 1<sub></sub>, hàm này nghịch biến trên

0; 1<sub></sub>
.


Dùng quy nạp, chứng minh dãy

<i>b</i><sub>2</sub><i><sub>n</sub></i><sub></sub><sub>1</sub>

là dãy giảm bị chặn dưới
bởi 2 2 và dãy

<i>b</i><sub>2</sub><i><sub>n</sub></i>

là dãy tăng và bị chặn trên bởi 3 3 2 1.



2


 


Do đó hai dãy này tồn tại giới hạn hữu hạn.


2,0


Đặt lim<i>b</i><sub>2</sub><i><sub>n</sub></i><sub></sub><sub>1</sub> <i>x</i>, lim<i>b</i><sub>2</sub><i><sub>n</sub></i> <i>y</i>. Khi đó, ,<i>x y</i> nằm trong khoảng

0,1


.


và thỏa mãn hệ phương trình


1,0


2 2


2
2


(2 5) 2 11


2 2 5 2 12


<i>t</i>
<i>t</i>


<i>x</i> <i>x</i>


<i>x</i> <i>x</i>



    





 <sub></sub><sub></sub> <sub></sub>


   


<sub></sub> <sub></sub>


 





(<i><sub>a</sub></i> <sub></sub>1)<i>t</i> <sub></sub><i><sub>a</sub>t</i> <sub>  </sub>1 <i><sub>t</sub></i> 1.


2 <sub>2</sub> <sub>11</sub> <sub>9</sub> <sub>4 5</sub> 2 <sub>2</sub> <sub>20</sub> <sub>4 5</sub> <sub>0</sub>


<i>x</i>  <i>x</i>    <i>x</i>  <i>x</i>   


1 (2 5 1)


<i>x</i>


</div>
<span class='text_page_counter'>(25)</span><div class='page_container' data-page=25>

2


2



1
2


2 1


1 <sub>2</sub> 2 1


<i>x</i> <i>y</i>


<i>xy</i> <i>y</i> <i>y</i>


<i>y</i>


<i>xy</i> <i>x</i> <i>x</i>


<i>y</i> <i>x</i>
<i>x</i>


  


  <sub></sub> <sub></sub> <sub></sub>


 




 


  



 <sub></sub> <sub></sub> <sub></sub> <sub></sub>





.


Giải hệ này ta có nghiệm 1


2


<i>x</i><i>y</i>  . Do đó dãy có giới hạn là
1


lim .


2


<i>n</i>


<i>b</i> 


<b>3.a </b>


Cho tam giác <i>ABC</i> nhọn, khơng cân, nội tiếp đường trịn ( ).<i>O</i> Gọi


<i>H</i> là trực tâm của tam giác <i>ABC</i>.


a) Gọi ,<i>I K</i> lần lượt là trung điểm của <i>AB AC</i>, và<i>B C</i><sub>1</sub>, <sub>1</sub> lần lượt là


chân đường cao kẻ từ ,<i>B C</i> của tam giác <i>ABC</i>. Đường thẳng <i>IK</i>


cắt <i>B C</i><sub>1 1</sub> tại <i>U</i>, đường thẳng <i>OH</i> cắt <i>IK</i> tại .<i>V</i> Chứng minh
rằng <i>V</i> là trực tâm tam giác <i>AHU</i>.


<b>3,0 </b>


Ta có nên ta chỉ cần chứng minh hay


Ta có nằm trên đường tròn Euler (E) của tam giác


ABC.
Do đó


1,5


Suy ra có cùng phương tích với đường trịn đường kính AH


và đường trịn đường kính OA, nên AU vng góc với đường
nối tâm của hai đường tròn.


1,5


<i>V</i>
<i>O</i>


<i>U</i>


<i>C1</i>



<i>B1</i>


<i>K</i>
<i>I</i>


<i>H</i>


<i>C</i>
<i>B</i>


<i>A</i>


<i>UV</i> <i>AH</i> <i>AU</i> <i>HV</i>


.


<i>AU</i> <i>OH</i>


1, , ,1


<i>B C I K</i>


1

.

1

.



<i>UB UC</i>

<i>UI UK</i>



</div>
<span class='text_page_counter'>(26)</span><div class='page_container' data-page=26>

Từ đó


Do vậy ta có điều phải chứng minh.



<b>3.</b>
<b>b</b>


Gọi <i>M</i> là trung điểm của đoạn thẳng <i>AH</i>, <i>T</i> là giao điểm của
tiếp tuyến tại <i>A</i> của đường tròn ( )<i>O</i> với đường thẳng <i>BC</i>, <i>P</i> là
hình chiếu vng góc của <i>O</i> trên đường thẳng <i>TM</i>. Chứng minh


rằng trung điểm của đoạn thẳng <i>MP</i> nằm trên đường tròn <i>Euler</i>


của tam giác <i>ABC</i>.


 (<i>Chú ý:</i> Đường tròn <i>Euler</i> của tam giác <i>ABC</i>là đường tròn
qua 9 điểm gồm trung điểm các cạnh, chân đường cao và trung
điểm của các đoạn thẳng nối các đỉnh với trực tâm tam giác <i>ABC</i>


).


<b>3,0 </b>


Ta có và nên AMOD là hình bình hành,


suy ra , do đó vng góc . Lại có AH vng


góc DT nên M là trực tâm tam giác TAD.


Gọi TM cắt AD tại N, khi đó MN vng góc ND suy ra N nằm trên
đường tròn Euler của tam giác ABC.


1,5



Gọi đối xứng M qua N, khi đó thuộc đường trịn ngoại tiếp


tam giác TAD, hay thuộc đường trịn đường kính OT, suy ra


tại . Do đó


Từ đó <i>N</i> là trung điểm <i>MP</i> thuộc đường tròn Euler của tam giác
.


<i>ABC</i>


1,5


<b>4.a </b> Tìm tất cả các hàm :<i>f</i> <b></b><b></b> thỏa mãn <b>4,0 </b>


.


<i>AU</i> <i>OH</i>


<i>AM</i> <i>OD</i> <i>AM OD</i>||


|| MD


<i>AO</i> <i>MD</i> <i>TA</i>


<i>P</i>

<i>P</i>



<i>P</i>



</div>
<span class='text_page_counter'>(27)</span><div class='page_container' data-page=27>

 




2 2


( ( ) ) ( ), , *


<i>f xf y</i> <i>x</i> <i>xy</i> <i>f x</i> <i>x y</i><b></b> .


Trong (*) cho ta có , do đó


hàm song ánh trên R.


Trong (*) cho ta có suy ra hoặc


0.5


+) Nếu Trong (*) cho ta có


(2)


Giả sử , trong (2) cho ta có




Do đó mâu thuẫn nên giả sử sai.


Vậy


1,0


Trong cho ta có (3)



Trong (3) cho ta có vì


Từ (1) ta có .


Trong (4) thay bởi ta có


(5)


0.5


Từ (4) kết hợp ta suy ra


Ta có


(6)


Trong (5) thay bởi ta có




1,0


Từ (*) cho ta có Vì



Do đó


(8).



Từ (6) và (8) ta có


1,0


1


<i>x</i> 

<i><sub>f f y</sub></i>

<sub>( ( ) 1)</sub>

<sub></sub>

<sub> </sub>

<i><sub>y</sub></i>

<i><sub>f</sub></i>

2

<sub>(1) (1)</sub>



<i>f</i>


0



<i>x</i>

<i><sub>f</sub></i><sub>(0)</sub><sub></sub> <i><sub>f</sub></i>2<sub>(0)</sub> <i><sub>f</sub></i><sub>(0)</sub><sub></sub><sub>0</sub>


(0) 1.


<i>f</i> 


(0) 1.


<i>f</i> 

<i>y</i>

0



2 2


(

)

( )

(2).



<i>f x</i>

<i>x</i>

<i>f x</i>

 

<i>x</i>

<i>R</i>



( ) 0



<i>f a</i>  <i>x</i> <i>a</i>


2 2 2


( ) ( ) 0 ( ) 0.


<i>f a</i> <i>a</i>  <i>f a</i>  <i>f a</i>  <i>a</i> <i>a</i>   <i>a</i> <i>a</i>


(0) 0


<i>f</i>  <i>f</i>(0)1 <i>f</i>(0)1


(0) 0.


<i>f</i> 


(*)

<i>y</i>

0

<i><sub>f x</sub></i>

<sub>( )</sub>

2

<sub></sub>

<i><sub>f x</sub></i>

2

<sub>( )</sub>

<sub> </sub>

<i><sub>x</sub></i>

<i><sub>R</sub></i>

<sub>.</sub>



1


<i>x</i>  <i><sub>f</sub></i><sub>(1)</sub><sub></sub><i><sub>f</sub></i>2<sub>(1)</sub><sub></sub><i><sub>f</sub></i><sub>(1)</sub><sub></sub><sub>1</sub> <i><sub>f</sub></i><sub>(1)</sub><sub></sub><sub>0.</sub>


( ( ) 1) 1 (4)


<i>f f y</i>   <i>y</i>


<i>y</i> <i>f y</i>( )1


( 2) ( ) 2 (5).



<i>f y</i> <i>f y</i>   <i>y</i> <i>R</i>


(1) 1


<i>f</i> 


2


2 2 2


((

2) )

(

2)

( )

2

( )

4 ( )

4,

.



<i>f y</i>

<i>f y</i>

<i>f y</i>

<i>f y</i>

<i>f y</i>

  

<i>y</i>

<i>R</i>



2


2 2 2


((

2) )

(

2)

( )

2

( )

4 ( )

4,

.



<i>f y</i>

<i>f y</i>

<i>f y</i>

<i>f y</i>

<i>f y</i>

  

<i>y</i>

<i>R</i>



<i>y</i> <i>y</i>2


(

4)

( )

4

(7).



<i>f y</i>

<i>f y</i>

  

<i>y</i>

<i>R</i>



4



<i>y</i> 

<i><sub>f x</sub></i>

<sub>(4</sub>

<sub></sub>

<i><sub>x</sub></i>

2

<sub>)</sub>

<sub></sub>

<sub>4</sub>

<i><sub>x</sub></i>

<sub></sub>

<i><sub>f x</sub></i>

2

<sub>( )</sub>

<sub> </sub>

<i><sub>x</sub></i>

<i><sub>R</sub></i>

<sub>.</sub>



(4)

4



<i>f</i>



2 2 2 2


(( 2) ) ( 4 4) ( 4 ) 4 ( ) 4 4 .


<i>f y</i> <i>f y</i>  <i>y</i> <i>f y</i>  <i>y</i>  <i>f y</i>  <i>y</i>  <i>y</i> <i>R</i>


2 2 2 2


((

2) )

(

4

4)

(

4 )

4

( )

4

4

.



<i>f y</i>

<i>f y</i>

<i>y</i>

<i>f y</i>

<i>y</i>

 

<i>f y</i>

<i>y</i>

  

<i>y</i>

<i>R</i>



( )

.



</div>
<span class='text_page_counter'>(28)</span><div class='page_container' data-page=28>

Thử lại và kết luận hàm số thỏa mãn bài tốn.


<b>4.</b>
<b>b</b>


Tìm đa thức hệ số ngun ( )<i>P x</i> thỏa mãn tính chất (2 )<i>P</i> <i>n</i> chia hết


cho ,<i>n</i> với mọi số nguyên dương .<i>n</i> <b>2,0 </b>


Với mọi số nguyên dương và mọi nguyên tố ta có



. Do đó ta có
.


Lại có nên


1,0


Với mỗi cố định, cho đủ lớn ta suy ra Từ đây suy


ra với mọi số nguyên dương Điều này kéo theo


.


1,0


( )



<i>f x</i>

<i>x x</i>

 

<i>R</i>



,


<i>k</i> <i>p</i>


2<i>kp</i> <sub></sub>2 (mod )<i>k</i> <i><sub>p</sub></i>


(2 )<i>k</i> (2 ) (mod )<i>kp</i>


<i>P</i> <i>P</i> <i>p</i>



(2 )<i>kp</i> (2 )<i>kp</i>


<i>P</i> <i>kp</i> <i>P</i> <i>p</i> <i><sub>P</sub></i>(2 ) .<i>k</i> <sub></sub><i><sub>p</sub></i>


<i>k</i> <i>p</i> <i><sub>P</sub></i>(2 )<i>k</i> <sub></sub>0.


(2 )<i>k</i> 0


<i>P</i>  <i>k</i>.


( )

0



</div>
<span class='text_page_counter'>(29)</span><div class='page_container' data-page=29>

SỞ GIÁO DỤC VÀ ĐÀO TẠO


BÌNH PHƯỚC



<b>ĐỀ THI CHÍNH THỨC </b>


KỲ THI LẬP ĐỘI TUYỂN DỰ THI


CHỌN HỌC SINH GIỎI QUỐC GIA NĂM 2018



<b>Mơn: TỐN</b>


Thời gian:<b> 180 </b>phút (khơng kể thời gian giao đề)
Ngày thi: 24/10/2018


<b>Câu 1. </b><i>(4.0 điểm)</i><b> </b>


<b>1. Giải hệ phương trình sau trên tập số thực: </b>









2 3


2 3


2 3


3 ln 1 3


3 ln 1 3


3 ln 1 3


<i>x</i> <i>x</i> <i>x</i> <i>y</i> <i>x</i>


<i>y</i> <i>y</i> <i>y</i> <i>z</i> <i>y</i>


<i>z</i> <i>z</i> <i>z</i> <i>x</i> <i>z</i>


 <sub></sub> <sub></sub> <sub></sub> <sub></sub> <sub></sub> <sub></sub>





     






     





.


<b>2. Tìm tất cả các hàm :</b><i>f</i>  thỏa mãn điều kiện sau:


 



2

<sub>.</sub>

 

<sub>,</sub> <sub>,</sub>


<i>f x</i>  <i>f y</i> <i>y</i><i>x f x</i> <i>x y</i>.


<b>Câu 2. </b><i>(2.0 điểm)</i><b> Trong mặt phẳng, cho đường tròn </b>

 

<i>O</i> và hai điểm ,<i>B C</i> cố định
nằm trên đường trịn đó sao cho <i>BC</i> khơng là đường kính. Xét một điểm <i>A</i> di
chuyển trên

 

<i>O</i> sao cho <i>AB</i> <i>AC</i> và <i>A</i> không trùng với ,<i>B C</i>. Gọi <i>D</i> và <i>E</i>


lần lượt là giao điểm của đường thẳng <i>BC</i> với đường phân giác trong và đường
phân giác ngồi của góc <i>BAC</i>. Gọi <i>I</i> là trung điểm của <i>DE</i>. Đường thẳng đi
qua trực tâm của tam giác <i>ABC</i> và vng góc với <i>AI</i>, cắt các đường thẳng <i>AD</i>


và <i>AE</i> tương ứng tại <i>M</i> và <i>N</i>. Chứng minh rằng đường thẳng <i>MN</i> luôn đi


qua một điểm cố định.



<b>Câu 3. </b><i>(7.0 điểm)</i><b> Cho </b><i>c</i>0 thỏa mãn

<i>n</i> 2

<i>c</i>, <i>n</i> *


<i>n</i>


   . Chứng minh 1


2 2


<i>c</i>
Với

 

<i>x</i> <i>x</i><sub>  </sub> <i>x</i> ,  <sub> </sub><i>x</i> là phần nguyên của <i>x</i>.


<b>Câu 4. </b><i>(7.0 điểm)</i><b> Cho </b><i>BP</i>

 

1 1 và với mỗi số <i>k</i>*,<i>k</i>2, gọi <i>BP k</i>

 

là tích các


ước nguyên tố của <i>k</i>. Hãy chỉ ra với mỗi số nguyên dương <i>N</i>, ta xây dựng


</div>
<span class='text_page_counter'>(30)</span><div class='page_container' data-page=30>

<b>ĐÁP ÁN – THANG ĐIỂM </b>



<b>Câu </b> <b>Nội dung </b> <b>Điểm </b>


<b> 1.1 </b> Giải hệ phương trình trên tập số thực:








2 3


2 3



2 3


3 ln 1 3


3 ln 1 3


3 ln 1 3


<i>x</i> <i>x</i> <i>x</i> <i>y</i> <i>x</i>


<i>y</i> <i>y</i> <i>y</i> <i>z</i> <i>y</i>


<i>z</i> <i>z</i> <i>z</i> <i>x</i> <i>z</i>


 <sub></sub> <sub></sub> <sub></sub> <sub></sub> <sub></sub> <sub></sub>





     





     






<b>2.0 </b>


Ta đưa hệ đã cho về dạng:








3 2


3 2


3 2


3 3 ln 1


3 3 ln 1


3 3 ln 1


<i>x</i> <i>x</i> <i>x</i> <i>x</i> <i>y</i>


<i>y</i> <i>y</i> <i>y</i> <i>y</i> <i>z</i>


<i>z</i> <i>z</i> <i>z</i> <i>z</i> <i>x</i>


 <sub></sub> <sub></sub> <sub></sub> <sub></sub> <sub></sub> <sub></sub>






     





     





Xét <i>f t</i>

 

<i>t</i>33<i>t</i>3 ln

<i>t</i>2  <i>t</i> 1 ,

<i>t</i>


Hệ trở thành:


 


 


 


<i>f x</i> <i>y</i>
<i>f y</i> <i>z</i>
<i>f z</i> <i>x</i>


 














0.25


0.25


Ta có

 



2
2


2


3 2


' 3 0,


1


<i>t</i> <i>t</i>


<i>f t</i> <i>t</i> <i>t</i>


<i>t</i> <i>t</i>
 



    


   nên hàm <i>f</i> đồng biến trên 


Khơng mất tính tổng quát, giả sử <i>z</i><i>y</i><i>x</i>, <i>x y z</i>, , (trường hợp


<i>x</i><i>y</i><i>z</i> tương tự)


Do <i>f</i> đồng biến trên  nên <i>f z</i>

 

 <i>f y</i>

 

<i>x</i><i>z</i>


Suy ra <i>x</i><i>y</i> <i>z</i> nên ta được


 

3

2



2 3 ln 1 0


<i>f x</i> <i>x</i><i>x</i>  <i>x</i>  <i>x</i> <i>x</i> 


<i>(*HS lý luận kiểu khác để suy ra x</i><i>y</i> <i>z đúng thì vẫn cho đủ điểm)</i>


0.25


0.25
0.25


Đặt <i>g x</i>

 

<i>x</i>32<i>x</i>3 ln

<i>x</i>2<i>x</i>1 ,

<i>x</i>


 

2 2



2


2 1


' 3 0,


1


<i>x</i>


<i>g x</i> <i>x</i> <i>x</i> <i>g</i>


<i>x</i> <i>x</i>




     


   đồng biến trên . 0.25


Mà <i>g</i>

 

1 0<i>x</i>1 là nghiệm của phương trình <i>f x</i>

 

<i>x</i> <sub>0.25 </sub>


</div>
<span class='text_page_counter'>(31)</span><div class='page_container' data-page=31>

<b>1.2 Tìm tất cả các hàm :</b><i>f</i>  thỏa mãn điều kiện sau:


 



2

<sub>.</sub>

 

 

<sub>1 ,</sub> <sub>,</sub>


<i>f x</i>  <i>f y</i> <i>y</i><i>x f x</i> <i>x y</i>.



<b>2.0 </b>


Ta chứng minh <i>f</i> đơn ánh:


,


<i>x y</i>


  mà <i>f x</i>

 

 <i>f y</i>

 

thì từ (1) ta có:


 

2

 

2

 

 



. .


<i>y</i><i>x f x</i>  <i>f x</i>  <i>f y</i>  <i>f x</i>  <i>f x</i> <i>x</i><i>x f x</i> <i>x</i><i>y</i> 0.25


Đặt <i>f</i>

 

0 <i>a</i>, thay <i>x</i>0 vào (1) thì được: <i>f f y</i>

 

<i>y</i> (2)
Thay <i>y</i>0 vào (2): <i>f f</i>

 

0

0 <i>f a</i>

 

0


Thay <i>x</i><i>a y</i>; 0 vào (1): <i>f a</i>

2<i>a</i>

<i>a f a</i>.

 

0
Thay <i><sub>y</sub></i><sub></sub><i><sub>a</sub></i>2<sub></sub><i><sub>a</sub></i><sub> vào (2): </sub>




 

 



2 2 <sub>0</sub> <sub>0</sub> <sub>0</sub> <sub>0</sub>


<i>a</i> <i>a</i> <i>f f a</i> <i>a</i>  <i>f</i> <i>a</i><i>a</i>  <i>f</i> 



0.25


0.25
Thay <i>y</i>0 vào (1): <i>f x</i>

 

2 <i>x f x</i>.

 

 <i>f x f f x</i>

 

.

 

,  <i>x</i>  (3)


Thay <i>y</i>0; <i>x</i> bởi <i>f x</i>

 

vào (1):


 

.

 

 

2

 

0

 

2
<i>f x f f x</i>  <i>f</i> <i>f x</i>  <i>f</i>  <i>f</i> <i>f x</i> 


    (4)


0.25


0.25
Từ (3), (4) suy ra <i>f x</i>

 

2  <i>f</i><sub></sub>

<i>f x</i>

 

2<sub></sub>


 


Do <i>f</i> đơn ánh nên

 

 <sub></sub>

 

<sub></sub>

 



 


2 2


2 2


<i>f x</i> <i>f</i> <i>f x</i> <i>f x</i> <i>x</i> (5)


 



 


<i>f x</i> <i>x</i>


<i>f x</i> <i>x</i>


 


 


 



0.25
0.25


Ta chứng minh chỉ xảy ra 1 trong 2 khả năng <i>f x</i>

 

<i>x</i> hoặc


 

;


<i>f x</i>  <i>x</i>  <i>x</i> 


Thật vậy, giả sử với <i>x<sub>o</sub></i>0,<i>y<sub>o</sub></i> 0 nào đó ta có <i>f x</i>

 

<i><sub>o</sub></i>  <i>x<sub>o</sub></i>,


 

<i>o</i> <i>o</i>


<i>f y</i> <i>y</i>


 

 



 

 




      


 


       


 


 


2 2 2


2 2 2 2


2 2 2 2


2


.


do 5


4 0


<i>o</i> <i>o</i> <i>o</i> <i>o</i> <i>o</i> <i>o</i> <i>o</i> <i>o</i> <i>o</i>


<i>o</i> <i>o</i> <i>o</i> <i>o</i> <i>o</i> <i>o</i> <i>o</i> <i>o</i>


<i>o</i> <i>o</i>



<i>f y</i> <i>x</i> <i>f x</i> <i>f y</i> <i>y</i> <i>x f x</i> <i>y</i> <i>x</i>


<i>f y</i> <i>x</i> <i>y</i> <i>x</i> <i>y</i> <i>x</i> <i>y</i> <i>x</i>


</div>
<span class='text_page_counter'>(32)</span><div class='page_container' data-page=32>

Điều này vơ lí do <i>x<sub>o</sub></i> 0;<i>y<sub>o</sub></i>0


Vậy <i>f x</i>

 

<i>x</i>, <i>x</i>  và <i>f x</i>

 

 <i>x</i>, <i>x</i> 


Thử lại ta thấy hai hàm số trên thỏa mãn đề bài.


0.25


<b>2 </b> <sub>Trong mặt phẳng, cho đường tròn </sub>

<sub> </sub>

<i>O</i> và hai điểm ,<i>B C</i> cố định


nằm trên đường trịn đó sao cho <i>BC</i> khơng là đường kính. Xét một


điểm <i>A</i> di chuyển trên

 

<i>O</i> sao cho <i>AB</i> <i>AC</i> và <i>A</i> không trùng
với ,<i>B C</i>. Gọi <i>D</i> và <i>E</i> lần lượt là giao điểm của đường thẳng <i>BC</i>


với đường phân giác trong và đường phân giác ngồi của góc <i>BAC</i>


. Gọi <i>I</i> là trung điểm của <i>DE</i>. Đường thẳng đi qua trực tâm của tam
giác <i>ABC</i> và vng góc với <i>AI</i>, cắt các đường thẳng <i>AD</i> và <i>AE</i>


tương ứng tại <i>M</i> và <i>N</i>. Chứng minh rằng đường thẳng <i>MN</i> luôn


đi qua một điểm cố định.


<b>2.0 </b>



Gọi <i>X</i> là điểm đối xứng của <i>O</i> qua <i>BC</i> suy ra <i>X</i> cố định. 0.25


Ta có <i>OX</i><i>AH</i>


//


<i>OX AH</i>(vì cùng vng góc <i>BC</i>) nên <i>AOXH</i> là hình bình hành


Suy ra <i>AO HX</i>// (1)


0.25


0.5
Lại có

<i>CBDE</i>

  1 <i>ID</i>2 <i>IB IC</i>. mà <i>ID</i><i>IA</i><i>IA</i>2 <i>IB IC</i>. <sub>0.25 </sub>


A


N


I
C


H
D
O


X M


</div>
<span class='text_page_counter'>(33)</span><div class='page_container' data-page=33>

Vậy <i>IA</i> tiếp xúc

 

<i>O</i> <i>IA</i><i>OA</i> (2) 0.25

Từ

   

1 , 2 suy ra <i>XH</i> <i>AI</i>, mà <i>MN</i> đi qua <i>H</i> và vng góc <i>AI</i> nên


<i>M,N,X</i> thẳng hàng. Vậy <i>MN</i> đi qua <i>X</i> cố định. 0.5


<b>3 </b> <sub>Cho </sub><i><sub>c</sub></i><sub></sub><sub>0</sub><sub> thỏa mãn </sub>

<i><sub>n</sub></i> <sub>2</sub>

<i>c</i><sub>,</sub> <i><sub>n</sub></i> *


<i>n</i>


   . Chứng minh 1


2 2


<i>c</i>


Với

 

<i>x</i> <i>x</i><sub>  </sub> <i>x</i> ,  <sub> </sub><i>x</i> là phần nguyên của<i>x</i>. <b>7.0 </b>






  


 2 1  <sub>2</sub>0 <sub>1</sub> <sub>2</sub>2 <sub>1</sub>  2<sub>2</sub> <sub>1</sub>


2 1 <i>n</i> <i>C</i> <i><sub>n</sub></i> 2<i>C</i> <i><sub>n</sub></i> ... 2<i>nC</i> <i>n<sub>n</sub></i>




  



 2 <i>C</i>1<sub>2</sub><i><sub>n</sub></i> <sub>1</sub>2<i>C</i>3<sub>2</sub><i><sub>n</sub></i> <sub>1</sub>... 2 <i>nC</i>2<sub>2</sub><i><sub>n</sub>n</i> <sub>1</sub>1


        


2 1


0 2 2


2 1 2 1 2 1


2 1 <i>n</i> <i>C</i> <i><sub>n</sub></i> 2<i>C</i> <i><sub>n</sub></i> ... 2<i>nC</i> <i><sub>n</sub>n</i>




  


 2 <i>C</i>1<sub>2</sub><i><sub>n</sub></i> <sub>1</sub>2<i>C</i>3<sub>2</sub><i><sub>n</sub></i> <sub>1</sub>... 2 <i>nC</i>2<sub>2</sub><i>n<sub>n</sub></i> <sub>1</sub>1


Đặt <i>A</i><sub>2</sub><i><sub>n</sub></i><sub>1</sub> <i>C</i>0<sub>2</sub><i><sub>n</sub></i><sub>1</sub>2<i>C</i>2<sub>2</sub><i><sub>n</sub></i><sub>1</sub>... 2 <i>nC</i>2<sub>2</sub><i><sub>n</sub>n</i><sub>1</sub>,


1 3 2 1


2 1 2 1 2 2 1 ... 2 2 1


<i>n</i> <i>n</i>


<i>n</i> <i>n</i> <i>n</i> <i>n</i>


<i>B</i>  <i>C</i>   <i>C</i>    <i>C</i> 






  <sub></sub>
 

 

  



 <sub></sub> <sub></sub> <sub></sub>


2 1


2 1 2 1


2 1 2 1
2 1


2 1 2 1


2 1 2


, ,


2 1 2



<i>n</i>
<i>n</i> <i>n</i>
<i>n</i> <i>n</i>
<i>n</i>
<i>n</i> <i>n</i>
<i>B</i> <i>A</i>
<i>A</i> <i>B</i>
<i>B</i> <i>A</i>
1.0


Từ



2 1


2 1 2 1


2 1 2 1 2 1


2 1


2 lim 2


<i>n</i>


<i>n</i> <i>n</i>


<i>n</i> <i>n</i> <i>n</i>


<i>A</i> <i>A</i>



<i>B</i> <i>B</i> <i>B</i>




 


  




   


2 2 2


2<i>n</i> 1 2 2<i>n</i> 1 1 2 2<i>n</i> 1 2<i>n</i> 1 1


<i>A</i> <sub></sub>  <i>B</i> <sub></sub>    <i>B</i> <sub></sub>  <i>A</i> <sub></sub>  <sub></sub><i>B</i><sub>2</sub><i><sub>n</sub></i><sub>1</sub> 2<sub></sub> <i>A</i><sub>2</sub><i><sub>n</sub></i><sub>1</sub>


2.0


 



 



 1 <i>A</i><sub>2</sub><i><sub>n</sub></i> <sub>1</sub>  2<i>B</i><sub>2</sub><i><sub>n</sub></i> <sub>1</sub> <i>A</i><sub>2</sub><i><sub>n</sub></i> <sub>1</sub> 2<i>B</i><sub>2</sub><i><sub>n</sub></i> <sub>1</sub>


 





  <i>A</i><sub>2</sub><i><sub>n</sub></i> <sub>1</sub> 2<i>B</i><sub>2</sub><i><sub>n</sub></i> <sub>1</sub> <i>B</i><sub>2</sub><i><sub>n</sub></i> <sub>1</sub> 2


2 1



2 1 2 1


2 1 2 1


</div>
<span class='text_page_counter'>(34)</span><div class='page_container' data-page=34>

Mà 2 1


2 1
2 1 2 1


2 1


1 1


lim lim


2 <sub>2</sub> 2 2


<i>n</i>
<i>n</i>
<i>n</i> <i>n</i>
<i>n</i>
<i>B</i>
<i>A</i>
<i>B</i> <i>A</i>
<i>B</i>


 

 






Theo đề ra:



 


 <sub>2</sub> <sub>1</sub> <sub>2</sub> <sub>1</sub> 2 ,    1


2 2


<i>n</i> <i>n</i>


<i>c</i> <i>B</i> <i>B</i> <i>n</i> <i>c</i>


<b>4 </b> Cho <i>BP</i>

 

1 1 và với mỗi số <i>k</i>*,<i>k</i>2 gọi <i>BP k</i>

 

là tích các ước
nguyên tố của <i>k</i>. Hãy chỉ ra với mỗi số nguyên dương <i>N</i>, ta xây
dựng được dãy số <i>a<sub>n</sub></i><sub></sub><sub>1</sub> <i>a<sub>n</sub></i><i>BP a</i>

 

<i><sub>n</sub></i> , <i>n</i> * luôn chứa <i>N</i>số hạng
liên tiếp tạo thành một cấp số cộng.


<b>7.0 </b>


Để chứng minh bài toán, ta sử dụng hai bổ đề sau:


<i><b>Bổ đề 1</b>:</i> Trong dãy <i>BP a</i>

 

<sub>1</sub> ,<i>BP a</i>

 

<sub>2</sub> ,...mỗi số hạng là ước của số
hạng tiếp sau nó.


<i><b>Chứng minh. </b></i>



– Vì <i>BP a</i>

 

<i><sub>n</sub></i> <i>a<sub>n</sub></i><i>BP a</i>

 

<i><sub>n</sub></i> <sub></sub><i>a<sub>n</sub></i><i>BP a</i>

 

<i><sub>n</sub></i> <sub></sub><i>a<sub>n</sub></i><sub></sub><sub>1</sub> nên mọi thừa số
nguyên tố của <i>BP a</i>

 

<i><sub>n</sub></i> đều là ước của <i>a<sub>n</sub></i><sub></sub><sub>1</sub>.


– Suy ra <i>BP a</i>

<sub> </sub>

<i><sub>n</sub></i> <i>BP a</i>

<sub></sub>

<i><sub>n</sub></i><sub>1</sub>

<sub></sub>



0.5
0.5
Đặt

 


<i>n</i>
<i>n</i>
<i>n</i>
<i>a</i>
<i>b</i>
<i>BP a</i>


 và



 


1
<i>n</i>
<i>n</i>
<i>n</i>
<i>BP a</i>
<i>z</i>
<i>BP a</i>


 với *



<i>n</i>
 


Dễ dàng ta có: <sub>,</sub> <sub>,</sub> *


<i>n</i> <i>n</i>


<i>b z</i>   <i>n</i>  .
Mặt khác


 



 


 



 


1
1


1 1 1


1


<i>n</i> <i>n</i>


<i>n</i> <i>n</i> <i>n</i>


<i>n</i> <i>n</i>


<i>n</i>



<i>n</i>


<i>n</i> <i>n</i> <i>n</i>


<i>n</i>


<i>a</i> <i>BP a</i>


<i>a</i> <i>BP a</i> <i>BP a</i>


<i>a</i> <i>b</i>


<i>b</i>


<i>z</i>


<i>BP a</i> <i>BP a</i> <i>BP a</i>


<i>BP a</i>


  

 
   
0.5
0.5


 <i><b>Bổ đề 2</b>:</i> Với mỗi số nguyên dương <i>N</i>, luôn tồn tại số nguyên


dương <i>M</i> sao cho


 



1 ... 2 1 *


<i>M</i> <i>M</i> <i>M N</i>


<i>z</i> <i>z</i> <sub></sub>  <i>z</i> <sub></sub> <sub></sub> 
<i><b>Chứng minh. </b></i>


 Ta có ln tồn tại một vài số ngun tố <i>p</i>2<i>N</i> nên luôn tồn tại
số nguyên tố <i>m</i> đủ lớn sao cho <i>a<sub>m</sub></i>chia hết cho mọi số nguyên tố
nhỏ hơn 2<i>N</i>.


</div>
<span class='text_page_counter'>(35)</span><div class='page_container' data-page=35>

Ta sẽ xây dựng dãy bắt đầu từ số hạng <i>a<sub>m</sub></i>(coi như số hạng đầu
của dãy).


Gọi <i>M</i> là số lớn hơn <i>m</i> sao cho <i>b<sub>M</sub></i> nhỏ nhất (theo nguyên lí cực
hạn).


Theo định nghĩa <i>b<sub>M</sub></i> nhỏ nhất, ta có


1


1


1, 1.


<i>M</i>



<i>M</i> <i>M</i> <i>M</i>


<i>M</i>


<i>b</i>


<i>z</i> <i>z</i> <i>z</i>


<i>b</i>







    


Do đó, ta chứng minh Bổ đề 2 bằng phản chứng.


 Giả sử

 

* không đúng thì *


min


,


<i>k</i> <i>k</i>


  để <i>z<sub>M k</sub></i> <sub>1</sub> 1.



Vậy 1<i>k</i><i>N</i>1 và <i>z<sub>M</sub></i> <i>z<sub>M</sub></i><sub></sub><sub>1</sub> ...<i>z<sub>M k</sub></i><sub> </sub><sub>2</sub> 1. Suy ra


1 1


<i>M k</i> <i>M</i>


<i>b</i> <sub> </sub> <i>b</i> <i>k</i> .


 Ta có <i>z<sub>M k</sub></i><sub> </sub><sub>1</sub> là tích của các số nguyên tố là ước của <i>a<sub>M k</sub></i><sub></sub> nhưng
không là ước của <i>a<sub>M k</sub></i><sub> </sub><sub>1</sub>.


 Mặt khác, nếu <i>z<sub>M k</sub></i><sub> </sub><sub>1</sub> 2<i>N</i> thì




1 1 1


<i>M k</i> <i>M</i> <i>M k</i> <i>M k</i>


<i>z</i>   <i>BP a</i>  <i>z</i>   <i>a</i>   (vô lý) nên <i>zM k</i> 1 2 .<i>N</i>
 Vì thế


1


1 1


1 1


2 2



<i>M k</i> <i>M</i> <i>M</i> <i>M</i>


<i>M k</i> <i>M</i>


<i>M k</i> <i>M k</i>


<i>b</i> <i>b</i> <i>k</i> <i>b</i> <i>k</i> <i>b</i> <i>N</i>


<i>b</i> <i>b</i>


<i>z</i> <i>z</i> <i>N</i> <i>N</i>


 


   


    


    


Điều này mâu thuẫn với giả thiết <i>b<sub>M</sub></i> nhỏ nhất. Vậy bổ đề được
chứng minh.




1.0


0.5



0.5


0.5


0.5


 Áp dụng Bổ đề 2, với mỗi <i>N</i>, tồn tại một số nguyên dương <i>M</i>


thỏa mãn:


<i>M</i>

<i>M</i> 1

...

<i>M N</i> 1


<i>BP a</i> <i>BP a</i> <sub></sub>  <i>BP a</i> <sub></sub> <sub></sub>


 Vậy <i>a<sub>M</sub></i>,<i>a<sub>M</sub></i><sub></sub><sub>1</sub>,...,<i>a<sub>M N</sub></i><sub></sub> <sub></sub><sub>1</sub> là các số hạng liên tiếp của một cấp số cộng.
1.0


</div>
<span class='text_page_counter'>(36)</span><div class='page_container' data-page=36>

SỞ GIÁO DỤC VAØ ĐAØO TẠO


BÌNH THUẬN



<b>ĐỀ THI CHÍNH THỨC </b>


KỲ THI CHỌN ĐỘI TUYỂN HỌC SINH GIỎI


LỚP 12 THPT DỰ THI QUỐC GIA



NĂM 2018–2019



<b>Mơn: TỐN</b>


Thời gian:<b> 180 </b>phút (không kể thời gian giao đề)
Ngày thi: 19/10/2018



<b>Bài 1. (5</b><i> điểm</i>) Giải phương trình nghiệm nguyên:




3 3 2 2 <sub>4</sub> 2 2 <sub>1.</sub>
<i>x</i> <i>y</i> <i>x y</i><i>xy</i>  <i>x</i> <i>xy</i><i>y</i> 


<b>Bài 2. (5</b><i> điểm</i>) Cho , 0;


2


<i>x y</i><sub> </sub>  <sub></sub>


 . Chứng minh rằng:




 


  




 


2 2 2 2 2


2



1 1 1


sin sin 1 sin cos 1 cos 1


9


.


2 sin sin 2 sin 2 sin sin 2 cos


<i>x</i> <i>y</i> <i>x</i> <i>y</i> <i>x</i>


<i>x</i> <i>y</i> <i>x</i> <i>y</i> <i>x</i> <i>y</i>


<b>Bài 3. (5</b><i> điểm</i>) Cho tam giác <i>ABC</i> có <i>AB</i><i>AC</i> và nội tiếp đường tròn

 

<i>O</i> . Phân
giác trong góc <i>BAC</i> cắt

 

<i>O</i> tại điểm <i>D</i> khác <i>A</i>, lấy <i>E</i> đối xứng <i>B</i> qua <i>AD</i>,
đường thẳng <i>BE</i> cắt

 

<i>O</i> tại <i>F</i> khác <i>B</i>. Lấy điểm <i>G</i> di chuyển trên cạnh <i>AC</i>


(<i>G</i> khác ,<i>A C</i>), đường thẳng <i>BG</i> cắt

 

<i>O</i> tại <i>H</i> khác .<i>B</i> Đường thẳng qua <i>C</i>


song song <i>AH</i> cắt <i>FD</i> tại <i>I</i>. Đường tròn ngoại tiếp tam giác <i>BCG</i> cắt <i>EI</i> tại


hai điểm phân biệt ,<i>K L</i>. Chứng minh rằng đường trung trực đoạn thẳng <i>KL</i>


luôn đi qua một điểm cố định.


<b>Bài 4. (5</b><i> điểm</i>) Cho 2018 tập hợp mà mỗi tập chứa đúng 45 phần tử. Biết rằng hai


</div>
<span class='text_page_counter'>(37)</span><div class='page_container' data-page=37>

<b>ĐÁP ÁN KỲ THI </b>




<b>Lời giải tóm tắt </b> <b>Điểm </b>


<b>Bài 1. (5</b><i> điểm</i>) Giải phương trình nghiệm nguyên:




3 3 2 2 <sub>4</sub> 2 2 <sub>1.</sub>
<i>x</i> <i>y</i> <i>x y</i><i>xy</i>  <i>x</i> <i>xy</i><i>y</i> 


Nhận xét: <i>x</i><i>y</i> 0,5


2 2



2 <i>x</i> <i>y</i> 4<i>xy</i> 1


    0,5






      


3 3 2 2 <sub>4</sub> 2 2 <sub>1</sub>


<i>x</i> <i>y</i> <i>x y</i> <i>xy</i> <i>x</i> <i>xy</i> <i>y</i>




 <i>x</i>2<i>y</i>2 <i>x</i><i>y</i>4 4<i>xy</i>1 0,5



2 2



2 4<i>xy</i> 1 2 <i>x</i> <i>y</i> <i>x</i> <i>y</i> 4 4<i>xy</i> 1 <i>x</i> <i>y</i> 4


          <sub>1,5 </sub>


2 <i>x</i> <i>y</i> 4 <i>x</i> <i>y</i> 3; 4; 5


       <sub>0,5 </sub>


3


<i>x</i><i>y</i> (không thỏa mãn) 0,5


4


<i>x</i><i>y</i> (không thỏa mãn) <sub>0,5 </sub>


5


<i>x</i><i>y</i> tìm được <i>x</i>1;<i>y</i>4hoặc <i>x</i>4;<i>y</i>1. <sub>0,5 </sub>


<b>Bài 2. (5</b><i> điểm</i>) Cho , 0;


2


<i>x y</i><sub> </sub>  <sub></sub>


 . Chứng minh rằng:





 


  




 


2 2 2 2 2


2


1 1 1


sin sin 1 sin cos 1 cos 1


9


.


2 sin sin 2 sin 2 sin sin 2 cos


<i>x</i> <i>y</i> <i>x</i> <i>y</i> <i>x</i>


<i>x</i> <i>y</i> <i>x</i> <i>y</i> <i>x</i> <i>y</i>


Đặt <i>a</i>sin sin ,<i>x</i> <i>y b</i>sin cos ,<i>x</i> <i>y c</i>cos<i>x</i> thì , ,<i>a b c</i>0; <i>a</i>2 <i>b</i>2<i>c</i>2 1 1,0


Ta cần chứng minh




2 2 2


1 1 1 9


.
4


1 1 c 1 <i>ab ac bc</i>


<i>a</i>  <i>b</i>       0,5


Thật vậy,




 



 





2 2 2


1 1 1 1 1 1


1 1 c 1 <i>a b a c</i> <i>b c b a</i> <i>c a c b</i>


<i>a</i>  <i>b</i>            











2 <i>a b c</i>
<i>a b a c b c</i>


 


  


</div>
<span class='text_page_counter'>(38)</span><div class='page_container' data-page=38>

<i>a</i><i>b a</i>



<i>c b</i>



<i>c</i>

 

 <i>a</i><i>b</i><i>c</i>



<i>ab</i><i>ac</i><i>bc</i>

<i>abc</i>












         


    


1
9
8


9


<i>a</i> <i>b</i> <i>c</i> <i>ab</i> <i>ac</i> <i>bc</i> <i>a</i> <i>b</i> <i>c</i> <i>ab</i> <i>ac</i> <i>bc</i>


<i>a</i> <i>b</i> <i>c</i> <i>ab</i> <i>ac</i> <i>bc</i>



1,0


Nên




2 2 2


1 1 1 9


.
4


1 1 c 1 <i>ab ac bc</i>


<i>a</i>  <i>b</i>       1,0


Đẳng thức xảy ra khi và chỉ khi 1 arccos 1 ,


4


3 3


<i>a</i><i>b</i><i>c</i> <i>x</i> <i>y</i> <sub>0,5 </sub>


<b>Bài 3. </b>(5<i> điểm</i>) Cho tam giác<i>ABC</i>có <i>AB</i> <i>AC</i> và nội tiếp đường trịn


 

<i>O</i> . Phân giác trong góc <i>BAC</i> cắt

 

<i>O</i> tại điểm <i>D</i> khác <i>A</i>, lấy <i>E</i> đối
xứng <i>B</i> qua <i>AD</i>, đường thẳng <i>BE</i> cắt

 

<i>O</i> tại <i>F</i> khác <i>B</i>. Lấy điểm



<i>G</i> di chuyển trên cạnh <i>AC</i> (<i>G</i> khác ,<i>A C</i>), đường thẳng <i>BG</i> cắt

 

<i>O</i>


tại <i>H</i> khác .<i>B</i> Đường thẳng qua <i>C</i> song song <i>AH</i> cắt <i>FD</i> tại <i>I</i>.
Đường tròn ngoại tiếp tam giác <i>BCG</i> cắt <i>EI</i> tại hai điểm phân biệt


,


<i>K L</i>. Chứng minh rằng đường trung trực đoạn thẳng <i>KL</i> luôn đi qua


một điểm cố định.


Gọi giao điểm của đường thẳng <i>EI</i> và <i>BC</i> là <i>J</i>. 0,5


<i>DF</i> là trục đối xứng của <i>EC</i> 1,0


   


<i>CEJ</i><i>ECI</i><i>HAC</i><i>HBC</i> nên tứ giác <i>BGEJ</i> nội tiếp 1,5


Phép nghịch đảo <i>k CE CG CJ CB</i>. .


<i>C</i>


<i>N</i>   biến đường tròn (<i>BCG</i>) thành đường


thẳng <i>EJ</i> nên biến ,<i>K L</i> thành chính nó. 1,0


Do đó 2 2


<i>CK</i> <i>CL</i> <i>k</i> hay đường trung trực đoạn thẳng <i>KL</i> luôn đi



qua điểm <i>C</i> cố định. 1,0


<b>Bài 4. (5</b><i> điểm</i>) Cho 2018 tập hợp mà mỗi tập chứa đúng 45 phần tử. Biết


rằng hai tập tùy ý trong các tập này đều có đúng một phần tử chung.
Chứng minh rằng tồn tại phần tử thuộc tất cả 2018 tập hợp đã cho.
Lấy tập A tùy ý, trong A sẽ có phần tử a thuộc ít nhất 45 tập hợp khác.


Nếu không, số tập hợp không quá 45x44 + 1 = 1981. 1,0


</div>
<span class='text_page_counter'>(39)</span><div class='page_container' data-page=39>

Với tập <i>B</i> bất kì, nếu a khơng thuộc <i>B</i> thì với mỗi tập <i>A<sub>i</sub></i>

1 <i>i</i> 45



đều có phần tử <i>a<sub>i</sub></i> chung với B mà <i>a<sub>i</sub></i> <i>a</i>. 1,0


Thành ra <i>B</i> khơng có phần tử chung với <i>A</i>, nếu có thì phần tử chung
đó phải thuộc tập <i>A<sub>i</sub></i>

1 <i>i</i> 45

nào đó nên A và <i>A<sub>i</sub></i>

1 <i>i</i> 45

có 2
phần tử chung. (Vơ lí)


1,0


</div>
<span class='text_page_counter'>(40)</span><div class='page_container' data-page=40>

SỞ GIÁO DỤC VAØ ĐAØO TẠO


HAØ NAM



<b>ĐỀ THI CHÍNH THỨC </b>


KỲ THI CHỌN HỌC SINH GIỎI LỚP 12


VAØ THAØNH LẬP ĐỘI TUYỂN THAM DỰ KỲ THI



CHỌN HSG

QUỐC GIA NĂM 2018–2019




<b>Mơn: TỐN</b>


Thời gian:<b> 180 </b>phút (khơng kể thời gian giao đề)
<b>Câu 1 </b><i>(4,0 điểm)<b>.</b></i> Cho dãy số

 

<i>x<sub>n</sub></i> xác định bởi:


*


1 1 2 3 2017 2018


1 2 3 2017 2018


0; <i><sub>n</sub></i> <i><sub>n</sub></i> ... , .


<i>n</i> <i>n</i> <i>n</i> <i>n</i> <i>n</i>


<i>x</i> <i>x</i> <i>x</i> <i>n</i>


<i>x</i> <i>x</i> <i>x</i> <i>x</i> <i>x</i>




         


a) Với mỗi <i>n</i>*, đặt <i><sub>n</sub></i> <sub>2</sub>


<i>n</i>


<i>n</i>
<i>y</i>



<i>x</i>


 . Chứng minh dãy số

 

<i>y<sub>n</sub></i> có giới hạn hữu hạn


và tính giới hạn đó.


b) Tìm các số  để dãy

<i>nx</i><i><sub>n</sub></i>

có giới hạn hữu hạn và giới hạn là một số khác 0.


<b>Câu 2 </b><i>(4,0 điểm)</i><b>. Tìm tất cả hàm số :</b><i>f</i>  thỏa mãn


 



2 2

 

2 <sub>,</sub>

 

<sub>1</sub>


<i>f x</i>  <i>f</i> <i>y</i> <i>xf x</i> <i>y</i> <i>x y</i>


<b>Câu 3 </b><i>(4,0 điểm)<b>.</b></i> Cho k là số nguyên dương, chứng minh rằng (4k2<sub>-1)</sub>2<sub> có ước </sub>


nguyên dương dạng 12 kn -1(n là số nguyên dương) khi và chỉ khi k chia hết
cho 3.


<b>Câu 4</b><i>(4,0 điểm)<b>.</b></i>Cho tam giác ABC nhọn, không cân nội tiếp đường tròn (O). Các


đường cao AD, BE và CF cắt nhau tại H, M là trung điểm cạnh BC. Đường tròn
(J) ngoại tiếp tam giác AEF cắt đường tròn (O) tại điểm thứ hai là K (KA), AM
cắt đường tròn (J) tại điểm thứ hai là Q (Q A), đường thẳng EF cắt đường
thẳng AD tại P, đoạn thẳng PM cắt đường tròn (J) tại N.


a) Chứng minh rằng các đường thẳng KF, EQ và BC hoặc đồng quy hoặc song


song.


b) Chứng minh rằng đường tròn ngoại tiếp tam giác DMN và đường tròn ngoại
tiếp tam giác BNC tiếp xúc nhau.


<b>Câu 5 </b><i>(4,0 điểm)<b>. </b></i>Trong kỳ thi chọn HSG lớp 12 mơn tốn năm học 2018-2019 có 20


</div>
<span class='text_page_counter'>(41)</span><div class='page_container' data-page=41>

<b>HƯỚNG DẪN CHẤM MƠN TỐN </b>



<b>Câu </b> <b>Nơi dung </b> <b>Điểm </b>


<b>1. </b>


<i>(4,0 đ)</i>


a) Từ giả thiết suy ra


2 2 2


1 1 2


1 1


0 2 2


<i>n</i> <i>n</i> <i>n</i> <i>n</i> <i>n</i>


<i>n</i> <i>n</i>


<i>x</i> <i>x</i> <i>x</i> <i>x</i> <i>x</i>



<i>x</i> <i>x</i>


          


Suy ra         


2 2 2 2


1 2 1 2 ... 1 2 .


<i>n</i> <i>n</i> <i>n</i>


<i>x</i> <i>x</i> <i>x</i> <i>x</i> <i>n</i> Do đó lim<i>x<sub>n</sub></i> 


1,0


Xét <i>x<sub>n</sub></i>2<sub></sub><sub>1</sub><i>x<sub>n</sub></i>2 

<i>x<sub>n</sub></i><sub></sub><sub>1</sub><i>x<sub>n</sub></i>



<i>x<sub>n</sub></i><sub></sub><sub>1</sub><i>x<sub>n</sub></i>



  


<sub></sub>       <sub></sub>      <sub></sub>


 2 3 2017 2018 2 3 2017 2018


1 2 3 2017 2018 1 2 3 2017 2018


2 <i><sub>n</sub></i> ... ...


<i>n</i> <i>n</i> <i>n</i> <i>n</i> <i>n</i> <i>n</i> <i>n</i> <i>n</i> <i>n</i> <i>n</i>



<i>x</i>


<i>x</i> <i>x</i> <i>x</i> <i>x</i> <i>x</i> <i>x</i> <i>x</i> <i>x</i> <i>x</i> <i>x</i>


2 3 4 2018 2019 2 2016 2017


1 2 3 2017 2018 2 3 2017 2018


2 ... 1 ...


<i>n</i>


<i>n</i> <i>n</i> <i>n</i> <i>n</i> <i>n</i> <i>x</i> <i>n</i> <i>n</i> <i>n</i>


<i>x</i> <i>x</i> <i>x</i> <i>x</i> <i>x</i> <i>x</i> <i>x</i> <i>x</i>


   


<sub></sub>        <sub> </sub>      <sub></sub>


   


Suy ra lim

<i>x</i>2<i><sub>n</sub></i><sub></sub><sub>1</sub><i>x<sub>n</sub></i>2

2


1,0


Ta có

 



2 2 2 2 2 2 2



2


1 1 2 ... 2 1 1


<i>n</i> <i>n</i> <i>n</i> <i>n</i>


<i>n</i> <i>x</i> <i>x</i> <i>x</i> <i>x</i> <i>x</i> <i>x</i> <i>x</i>


<i>x</i>


<i>n</i> <i>n</i>


  


      




Áp dụng định lý trung bình Cesaro ta có


2 2

 

2 2

2 2

2


2


1 1 2 ... 2 1 1


lim<i>xn</i> lim <i>xn</i> <i>xn</i> <i>xn</i> <i>xn</i> <i>x</i> <i>x</i> <i>x</i> 2


<i>n</i> <i>n</i>



  


      


 


Do đó lim <sub>2</sub> 1


2


<i>n</i>


<i>n</i>
<i>x</i> 


1,0


b) Xét <i><sub>n</sub></i> <i><sub>n</sub></i> <sub>2</sub> <i><sub>n</sub></i> 2


<i>n</i>


<i>n</i>


<i>z</i> <i>nx</i> <i>x</i>


<i>x</i>


 



  <sub>0,5 </sub>


Từ đó:


+) Nếu   2 thì lim<i>z<sub>n</sub></i>  


+) Nếu   2 thì lim<i>z<sub>n</sub></i> 0


+) Nếu   2 thì lim 1


2


<i>n</i>


<i>z</i> 


Vậy   2 là giá trị cần tìm thỏa mãn đề bài.


0,5


<b>2. </b>


<i>(4,0 đ)</i>


Thay x = 0 vào (1) có <i>f</i>

<i>f</i>2

 

<i>y</i>

 <i>y</i>2.


Thay x = y = 0 vào (1) có <i>f</i>

<i>f</i>2

 

0

0 <i>a f a</i>:

 

0


</div>
<span class='text_page_counter'>(42)</span><div class='page_container' data-page=42>

Thay y = 0 vào (1) có <i>f x</i>

 

2 <i>xf x</i>

 

 <i>x</i> .
Thay x bởi (–x) có


 

 

 

 



 

 



        


     



2


0


<i>f x</i> <i>xf</i> <i>x</i> <i>x</i> <i>xf x</i> <i>xf</i> <i>x</i>


<i>f x</i> <i>f</i> <i>x</i> <i>x</i>


Mà f(0) = 0 suy ra f là hàm lẻ mà <i>f</i>

<i>f</i>2

 

<i>y</i>

<i>y</i>2 suy ra <i>f</i> nhận giá
trị trên , do đó f là tồn ánh.


1,0


 



 

<sub></sub>

 

<sub></sub>



 

 



2 2 2 2 <sub>,</sub>



, 0.


<i>f x</i> <i>f</i> <i>y</i> <i>f</i> <i>x</i> <i>f f</i> <i>y</i> <i>x y</i>


<i>f a b</i> <i>f a</i> <i>f b</i> <i>a b</i>


    


     




Thay <i>a</i> bởi <i>a + b</i> suy ra <i>f a b</i>

 <i>f a</i>

 

 <i>f b</i>

 

<i>a b</i>, 0.
Vì <i>f</i> lẻ suy ra <i>f a b</i>

 <i>f a</i>

 

 <i>f b</i>

 

<i>a</i>, bf: cộng tính.


1,0




 

 

 

 

 





 

 

  

 



 

 

 



2 <sub>2</sub>



2


1 2 1 2 1


1 1 1 1 1


1 . .


<i>f</i> <i>x</i> <i>f x</i> <i>f x</i> <i>xf x</i> <i>f x</i> <i>f</i>


<i>f</i> <i>x</i> <i>x</i> <i>f x</i> <i>x</i> <i>f x</i> <i>f</i>


<i>f x</i> <i>xf</i> <i>f x</i> <i>k x x</i>


      


      


     




Thử lại suy ra k=-1. Vậy <i>f x</i>

 

 <i>x</i>.


1,0


<b>3. </b>


<i>(4,0 đ)</i>



<i><b>Bổ đề: </b>x y</i>, ; 4x

<i>y</i>1 | 4x

2 1

2  <i>x</i><i>y</i>.


Chứng minh: Nếu x=y thì 4x<i>y</i> 1

4x21 | 4x

 

2 1

2.
Gọi (x;y) là cặp số nguyên dương “tồi” nếu

4x<i>y</i>1 | 4x

2 1

2
nhưng <i>x</i><i>y</i>.


<i>Nhận xét: </i>


i) Nếu (x; y) là cặp “tồi” và x<y thì tồn tại số nguyên dương z sao
cho z < x và (x; z) là cặp số “tồi”.


Thật vậy: đặt 




2
2


4x 1


4x 1


<i>r</i>


<i>y</i>


  



<i>r</i> <i>r</i> 1  <i>r</i> 4x<i>y</i>1  

4x2 1

2  1(mod 4x)


4xz 1, .



<i>r</i> <i>z</i> 


   


2

2


2


4x 1


4xz 1 4x 1


4x 1


<i>x</i> <i>y</i>


<i>y</i>


     


 <i>z</i><i>x</i>


</div>
<span class='text_page_counter'>(43)</span><div class='page_container' data-page=43>

4xz 1 | 4x

2 1

2 suy ra

<i>x z</i>;

là cặp ‘’tồi’’.
ii) Nếu <i>(x; y)</i> là cặp “tồi” thì <i>(y; x)</i> cũng là cặp “tồi”.


Thật vậy:


 




<sub></sub>

<sub></sub>



<sub></sub>

<sub></sub>



  


      


  


2
2


2


2 <sub>2</sub> 2


2 2 4 2


2
2


1 1 4x mod (4x 1 )


4 1 4 4x 16 4x 1 0 mod (4x 1 )


4x 1 | 4 1


<i>y</i> <i>y</i>



<i>y</i> <i>y</i> <i>y</i> <i>y</i> <i>y</i>


<i>y</i> <i>y</i>



1,0


Giả sử tồn tại ít nhất 1 cặp “tồi”. Gọi (<i>x; y</i>) là cặp “tồi” sao cho
(2<i>x + y</i>) nhỏ nhất.


Nếu <i>x</i> < <i>y</i>, theo (i) suy ra tồn tại (<i>x; z</i>) là cặp “tồi” sao cho
2<i>x </i>+ z < 2x + <i>y</i>.


Nếu <i>y</i> < <i>x</i>, theo (ii) suy ra (<i>y</i>, <i>x</i>) là cặp “tồi” và 2<i>y </i>+ <i>x </i>< 2<i>x </i>+ <i>y</i>.
Vậy <i>x </i>= <i>y</i>.


1,0


Áp dụng bổ đề với <i>x</i> = <i>k</i>, <i>y</i> = 3<i>n</i>.


 2  2  


(12<i>kn</i> 1)|(4<i>k</i> 1) <i>k</i> 3 .<i>n</i> 1,0


<b>4. </b>
(<i>4,0 đ)</i>


Không mất tổng quát giả sử <i>C QSDE</i>

  1

<i>QFBA</i>

 1



<b>a) Gọi </b><i>A</i>’ là điểm đối xứng với <i>H</i> qua <i>M</i>, suy ra <i>BHCA</i>’ là hình bình
hành. Do đó <i>NF</i>2 <i>NB NA</i>. suy ra  là đường kính của
đường tròn ngoại tiếp tam giác <i>ABC</i>. Suy ra


</div>
<span class='text_page_counter'>(44)</span><div class='page_container' data-page=44>

 



  2  2  2 2  2  2


/( ) /( )


<i>N O</i> <i>M O</i>


<i>P</i> <i>P</i> <i>NO</i> <i>R</i> <i>MO</i> <i>R</i> <i>NO</i> <i>MO</i>


Dễ thấy <i>AH</i> là đường kính của đường trịn (J), suy ra . Từ (1)
và (2) suy ra <i>K, H, A’</i> thẳng hàng. Mà<i> H, M, A’ </i>thẳng hàng nên
suy ra <i>K, H, M, A’</i> thẳng hàng.


Gọi L là giao điểm của <i>AK</i> và <i>BC</i>.


Từ các kết quả trên và giả thiết, suy ra <i>H</i> là trực tâm của các tam
giác <i>ALM</i>.
suy ra LH vuông góc với AM, gọi




 





2 2 2 2


0



0


2 . 0 ( ).


<i>NO</i> <i>NI</i> <i>MO</i> <i>MI</i>


<i>NO NI</i> <i>NO</i> <i>NI</i> <i>MO</i> <i>MI</i> <i>MO</i> <i>MI</i>


<i>IO NM</i> <i>MN</i> <i>IO đpcm</i>


 


  <sub></sub>  <sub></sub>


 


      


   


   


       


  suy ra


các tứ giác: ABDE, ALDQ nội tiếp, suy ra <i>DB</i><i>DC</i> nội tiếp.


Ta có:    



2 2


<i>B</i> <i>B</i>


<i>DBI</i> <i>DBC</i> <i>BAD</i> <i>DIB</i> <i>DBI</i>. Suy ra các tứ giác


KLBF, CMQE nội tiếp


Như vậy: <i>LB</i> là trục đẳng phương của hai đường tròn (<i>LBQE</i>) và


<i>KLBF</i>); <i>KF</i> là trục đẳng phương của hai đường tròn ((<i>KLBF</i>) và (<i>J</i>);


<i>EQ</i> là trục đẳng phương của hai đường tròn (<i>J</i>) và (<i>LBQE</i>).


Do đó 3 đường thẳng <i>KF, EQ</i> và <i>BC </i>đồng quy hoặc song song.
1,0


<b>b) Ta có: </b><i>AK</i> là trục đẳng phương của hai đường tròn (<i>O</i>) và (<i>J</i>); <i>EF </i>


là trục đẳng phương của hai đường tròn (<i>J</i>) và (<i>BFEC</i>); <i>BC</i> là trục
đẳng phương của hai đường tròn (<i>BFEC</i>) và (<i>O</i>), mà <i>AK</i> cắt <i>BC</i>


tại <i>L</i>, suy ra <i>AK, EF, BC</i> đồng quy tại <i>L</i>.


Ta có <i>M</i> là tâm của đường tròn (<i>BFEC</i>), suy ra <i>AB</i><i>AC</i>2<i>BC</i>


kết hợp với


    



. . . <i>CA GB</i>. 1 <i>BA GC</i>. (1)


<i>CA GB</i> <i>CB GA</i> <i>BA GC</i>


<i>CB GA</i> <i>BC GA</i> .


Suy ra P là trực tâm tam giác <i>JLM</i>. Do đó




  





  




( ) sin


. ,


( ) sin


( ) sin


. (2)


( ) sin



<i>CA</i> <i>FA</i> <i>S GAF</i> <i>GA</i> <i>AGD</i>


<i>CB</i> <i>FB</i> <i>S GBF</i> <i>GB</i> <i>BGD</i>


<i>BA</i> <i>EA</i> <i>S GEA</i> <i>GA</i> <i>AGD</i>


<i>BC</i> <i>EC</i> <i>S GEC</i> <i>GC</i> <i>CGD</i>


Gọi <i>S</i> là giao điểm của <i>JL</i> và <i>MP</i>, ta có tứ giác <i>LDPS</i> nội tiếp, suy
ra <i>CA</i> <i>GA DA BA</i>. ;  <i>GA DA</i>.


<i>CB</i> <i>GB DB BC</i> <i>GC DC</i>


</div>
<span class='text_page_counter'>(45)</span><div class='page_container' data-page=45>

 <i>CA GB</i>.  <i>DA BA GC</i>; .  <i>DA</i> (3)


<i>CB GA</i> <i>DB BC GA</i> <i>DC</i>


Xét tứ giác toàn phần <i>AEHFBC</i>, ta có (<i>A,H,P,D</i>) = –1, mà <i>J</i> là


trung điểm <i>AH</i>, suy ra <i>DA</i> 1 <i>DA</i> 1 1 1 (<i>dfcm</i>)


<i>DB</i>   <i>DC</i>  <i>DB</i> <i>DA</i> <i>DC</i>


(4). Từ (3) và (4) suy ra <i>B A C</i><sub>1</sub> <sub>1 1</sub><i>B HC</i><sub>1</sub> <sub>1</sub> <i>BAC</i>, mà <i>A B C</i><sub>1 1 1</sub> suy
ra LN là tiếp tuyến của (<i>J</i>). Suy ra <i>ABC</i> là tiếp tuyến của đường


tròn (<i>BNC</i>) (5).
Từ <i>A B C</i><sub>1 1 1</sub>4 điểm <i>A, K, D, M</i> cùng thuộc một đường tròn, suy ra



1 ; 1 ; 1


<i>A x B y C z</i> là tiếp tuyến của đường tròn ngoại tiếp tam giác


<i>MND</i> (6).
Từ (5) và (6) suy ra hai đường tròn (<i>BNC</i>) và (<i>MND</i>) tiếp xúc
nhau tại <i>N</i> (đpcm).


1,0


<b>5. </b>
(<i>4,0 đ)</i>


Ta gọi chung một nhóm “tình bạn” hoặc một nhóm “xa lạ” là một
nhóm “đặc biệt”. Số nhóm 3 người mà khơng phải nhóm “đặc biệt”
là <i>C</i>3<sub>20</sub><i>n</i>1140<i>n</i>.


1,0
Ta gọi T là số bộ ( ,{ , })<i>A B C</i> sao cho A quen cả B và C hoặc A không
quen cả B và C.


<i><b>TH1:</b></i> Với mỗi nhóm đặc biệt số bộ ( ,{ , })<i>A B C</i> là 3, vậy số bộ
( ,{ , })<i>A B C</i> như trên là 3n.


0,5


<i><b>TH2:</b></i> Với mỗi nhóm 3 người, mà khơng phải nhóm đặc biệt số bộ


( ,{ , })<i>A B C</i> như trên là 1 , trường hợp số bộ ( ,{ , })<i>A B C</i> là



3


20 1140


<i>C</i> <i>n</i> <i>n</i>.Vậy <i>T</i>11402<i>n</i>.


0,5
Với mỗi bạn học sinh <i>A</i>, giả sử bạn A quen với <i>x</i> học sinh và
không quen với <i>y</i> học sinh. Ta có <i>x</i><i>y</i>19 và số bộ ( ,{ , })<i>A B C</i>


ứng với học sinh A này là


 





2 2 1 1 1 19 18


2 2 2 2 81


<i>x</i> <i>y</i>


<i>C</i> <i>C</i>  <i>x x</i>  <i>y y</i>  <i>x x</i>  <i>x</i> <i>x</i>  .


0,5


Suy ra <i>T</i>81.201620 hay 11402<i>n</i>1620. Vậy <i>n</i>240. 0,5
Để chỉ ra dấu bằng, ta đánh số các bạn học sinh là <i>A</i><sub>1</sub>,,<i>A</i><sub>20</sub> và xây
dựng một mơ hình như sau: <i>A<sub>i</sub></i> và <i>A<sub>j</sub></i> quen nhau nếu ,<i>i j</i> cùng tính
chẵn lẻ. Và <i>A<sub>i</sub></i> không quen <i>A<sub>j</sub></i> nếu ,<i>i j</i> khác chẵn lẻ. Khi đó số bộ
( ,{ , })<i>A B C</i> ứng với một nhà toán học <i>A</i> sẽ là <i>C</i><sub>10</sub>2 <i>C</i><sub>9</sub>2 81, tức dấu
bằng đã xảy ra.



</div>
<span class='text_page_counter'>(46)</span><div class='page_container' data-page=46>

SỞ GIÁO DỤC VAØ ĐAØO TẠO


HÀ NỘI



<b>ĐỀ THI CHÍNH THỨC </b>


KỲ THI CHỌN ĐỘI TUYỂN HỌC SINH GIỎI


THAØNH PHỐ LỚP 12 THPT NĂM 2018–2019



<b>Mơn: TỐN</b>


Thời gian:<b> 180 </b>phút (không kể thời gian giao đề)
Ngày thi: 28/9/2018


<b>Bài 1. </b><i>(4 điểm)</i> Gọi <i>d d</i><sub>1</sub>, <sub>2</sub>,...,<i>d<sub>k</sub></i> là tất cả các ước nguyên dương của <i>n</i> được sắp xếp
theo thứ tự tăng dần. Tìm tất cả các số nguyên dương <i>n</i> có tích chất sau:


5 3


5 7


40
.


7 8 3


<i>d</i> <i>d</i>


<i>d</i> <i>d</i> <i>n</i>



  





 







<b>Bài 2. </b><i>(4 điểm)</i><b> Cho đa thức </b> <i>P x</i>

 

<i>xp</i><i>ax</i>2 <i>bx c</i> trong đó , ,<i>a b c</i> là các số
nguyên và <i>p</i> là số nguyên tố. Biết rằng <i>P x</i>

 

có ba nghiệm nguyên <i>x x x</i><sub>1</sub>, <sub>2</sub>, <sub>3</sub>
thỏa mãn

<i>x</i><sub>1</sub><i>x</i><sub>2</sub>



<i>x</i><sub>2</sub> <i>x</i><sub>3</sub>



<i>x</i><sub>3</sub> <i>x</i><sub>1</sub>

không chia hết cho <i>p</i>. Chứng minh rằng


<i>abc</i><i>ac</i> chia hết cho <i>p</i>3.


Bài 3. <i>(4 điểm)</i> Với mỗi số nguyên dương <i>n</i> xét hàm số


 

2<i>n</i> 2<i>n</i> 1 <sub>...</sub> <sub>1.</sub>


<i>n</i>


<i>f x</i> <i>x</i> <i>x</i>   <i>x</i>


1) Chứng minh rằng hàm <i>f x<sub>n</sub></i>

 

luôn đạt giá trị nhỏ nhất tại một điểm <i>x<sub>n</sub></i> duy
nhất.


2) Tìm lim <i>f x<sub>n</sub></i>

 

<i><sub>n</sub></i> .


<b>Bài 4. </b><i>(4 điểm)</i> Cho hai đường tròn

 

<i>O</i> và

 

<i>O</i>' cắt nhau tại <i>A</i> và <i>B</i>. Qua <i>A</i> kẻ
hai đường thẳng <sub>1</sub> và <sub>2</sub>. <sub>1</sub> cắt hai đường tròn

 

<i>O</i> và

 

<i>O</i>' lần lượt tại <i>C</i>


và ;<i>D</i> <sub>2</sub> cắt hai đường tròn

 

<i>O</i> và

 

<i>O</i>' lần lượt tại <i>E</i> và <i>F</i> ( , , ,<i>C D E F</i> khác


<i>A</i>). Các đường trung trực của <i>CD</i> và <i>EF</i>cắt nhau tại <i>K</i>. Đường thẳng <i>d</i> thay
đổi đi qua <i>K</i> cắt đường tròn

 

<i>O</i>' tại , .<i>P Q</i> <sub> Chứng minh rằng trực tâm tam giác </sub>


<i>APQ</i> ln nằm trên một đường trịn cố định.


<b>Bài 5. </b><i>(4 điểm) </i>Xét các số hữu tỉ dương <i>x x</i><sub>1</sub>, <sub>2</sub>,...,<i>x<sub>n</sub></i> thỏa mãn


1 2


1 2


1 1 1


; ; ...; <i><sub>n</sub></i>


<i>n</i>


<i>x</i> <i>x</i> <i>x</i>


<i>p</i> <i>p</i> <i>p</i>


  


là các số nguyên dương (với 1. ...2 <i>n</i> <sub>,</sub>

<sub></sub>

<sub>1; 2; ...;</sub>

<sub></sub>



<i>i</i>


<i>i</i>


<i>x x</i> <i>x</i>


<i>p</i> <i>i</i> <i>n</i>


<i>x</i>


   ).


1) Chứng minh <i>x x</i><sub>1 2</sub>...<i>x<sub>n</sub></i> 1.


</div>
<span class='text_page_counter'>(47)</span><div class='page_container' data-page=47>

<b>HƯỚNG DẪN CHẤM </b>



<b>Bài </b> <b>Nội dung </b> <b>Điểm </b>


<b>1. </b>


<i>(4 đ)</i>


<i>TH1:</i> Nếu n lẻ thì <i>d<sub>i</sub></i>lẻ với mọi <i>i</i>

1, 2,...,<i>k</i>



Từ <sub>5</sub> <sub>7</sub> 5 7 5 7


7 5 7 5


|8 |



7 8 3


|7 |7


<i>d</i> <i>d</i> <i>d</i> <i>d</i>


<i>d</i> <i>d</i> <i>n</i>


<i>d</i> <i>d</i> <i>d</i> <i>d</i>


 


 


  <sub></sub> <sub></sub>


 


 


0,5


7 7 5 21 5 1, 3,7, 21


<i>d</i> <i>d</i> <i>n</i> <i>d</i>


     là các ước của n. 0,5


Mà theo giả thiết <i>d</i><sub>5</sub> 40<i>d</i><sub>3</sub> 40<i>d</i><sub>1</sub> 1,<i>d</i><sub>2</sub> 3,<i>d</i><sub>3</sub> 7,<i>d</i><sub>4</sub> 21 <sub>0,5 </sub>



5 47 21.47 987


<i>d</i> <i>n</i>


    


Thử lại thấy <i>n</i>987thỏa mãn yêu cầu bài toán 0,5


+ Nếu n chẵn suy ra <i>d</i><sub>1</sub> 1,<i>d</i><sub>2</sub> 2.Từ 7<i>d</i><sub>5</sub>8<i>d</i><sub>7</sub> 3<i>n</i>chẵn suy ra <i>d</i><sub>5</sub>


chẵn nên <i>d</i><sub>3</sub> <i>d</i><sub>5</sub>40 chẵn. Suy ra <i>d</i><sub>3</sub> 4hoặc <i>d</i><sub>3</sub> 2 ,<i>k k</i>2 1,0


Nếu <sub>3</sub>


2 3


|


2 , 2


2


<i>k n</i>


<i>d</i> <i>k k</i>


<i>d</i> <i>k</i> <i>d</i>






  <sub> </sub>


  





điều này vô lý.


Vậy <i>d</i><sub>3</sub> 4<i>d</i><sub>5</sub> 4411, 22 là hai ước của <i>n</i> mà hai ước này
nằm giữa <i>d d</i><sub>3</sub>, <sub>5</sub>điều này vô lý vậy n chẵn không thỏa mãn
Kết luận: <i>n</i>987là số tự nhiên duy nhất thỏa mãn yêu cầu
bài toán


1,0


<b>2. </b>


<i>(4 đ) </i>


Ta có

 



1


2 2


1 1 1 1 1 1 1 1 0


<i>p</i>


<i>p</i>


<i>P x</i> <i>x</i> <i>ax</i> <i>bx</i> <i>c</i> <i>x</i> <i>x</i> <i>ax</i>  <i>b</i> <i>x</i> <i>c</i> 0,5


Mà theo định lý Fecma ta có



1 1 0 mod


<i>p</i>


<i>x</i> <i>x</i>  <i>p</i> suy ra




2


1 1 1 0 mod


<i>ax</i>  <i>b</i> <i>x</i>  <i>c</i> <i>p</i>


0,5


Tương tự ta có <i>ax</i>2<sub>2</sub> 

<i>b</i>1

<i>x</i><sub>2</sub> <i>c</i> 0 mod

<i>p</i>





2


3 1 3 0 mod



<i>ax</i>  <i>b</i> <i>x</i>  <i>c</i> <i>p</i> 0,5


Suy ra

<i>ax</i><sub>1</sub>2 

<i>b</i>1

<i>x</i><sub>1</sub><i>c</i>

<i>ax</i><sub>2</sub>2

<i>b</i>1

<i>x</i><sub>2</sub><i>c</i>

0 mod

<i>p</i>



<sub></sub>

<i>x</i><sub>1</sub><i>x</i><sub>2</sub>

<sub> </sub>

<sub></sub><i>a x</i><sub>1</sub><i>x</i><sub>2</sub>

<sub> </sub>

 <i>b</i>1

<sub></sub>

<sub></sub>0 mod

<sub></sub>

<i>p</i>

<sub></sub>



1,0


<i>x</i><sub>1</sub><i>x</i><sub>2</sub>



<i>x</i><sub>2</sub> <i>x</i><sub>3</sub>



<i>x</i><sub>3</sub><i>x</i><sub>1</sub>

không chia hết cho <i>p</i> suy ra


1 2

 

1

0 mod



<i>a x</i> <i>x</i> <i>b</i> <i>p</i>


    


 


</div>
<span class='text_page_counter'>(48)</span><div class='page_container' data-page=48>

Tương tự <sub></sub><i>a x</i>

<sub>2</sub><i>x</i><sub>3</sub>

 

 <i>b</i>1

<sub></sub>0 mod

<i>p</i>



Suy ra <sub></sub><i>a x</i>

<sub>1</sub><i>x</i><sub>2</sub>

 

 <i>b</i>1

<sub></sub><sub></sub><i>a x</i>

<sub>2</sub> <i>x</i><sub>3</sub>

 

 <i>b</i>1

<sub></sub> 0 mod

<i>p</i>



1 2

0 mod

0 mod



<i>a x</i> <i>x</i> <i>p</i> <i>a</i> <i>p</i>


    


0,5



<i>b</i> 1

0 mod

<i>p</i>

<i>c</i> 0 mod

<i>p</i>



    


<i>abc</i> <i>ac</i>


  chia hết cho <i>p</i>3 0,5


<b>3.</b>


<i>(4 đ) </i>


<b>1) Với </b><i>x</i>0<i>xk</i> 0  <i>k</i>

1, 2,..., 2<i>n</i>

 <i>f x<sub>n</sub></i>( ) 1
Với <i>x</i>  1 <i>x</i>2<i>k</i>1(<i>x</i>1)0  <i>k</i>

1, 2,....,<i>n</i>






 

2 1   


1


( ) ( 1) 1 1


<i>n</i>
<i>k</i>
<i>n</i>


<i>k</i>



<i>f x</i> <i>x</i> <i>x</i>


Do vậy để tìm giá trị nhỏ nhất của hàm số <i>f x<sub>n</sub></i>( ) ta chỉ cần xét
1,0


<i>x</i> <sub></sub> <sub></sub> là đủ


0,5


Với <i>x</i> <sub></sub> 1,0<sub></sub> ta có


2 1 2 1 2


/


2


1 2 (2 1) 1


( ) ( )


1 (1 )


<i>n</i> <i>n</i> <i>n</i>


<i>n</i> <i>n</i>


<i>x</i> <i>nx</i> <i>n</i> <i>x</i>


<i>f x</i> <i>f x</i>



<i>x</i> <i>x</i>


 


   


  


 


Đặt <i>g x<sub>n</sub></i>( )2<i>nx</i>2<i>n</i>1(2<i>n</i>1)<i>x</i>2<i>n</i>1


 <sub></sub> <sub></sub>


<i>g x<sub>n</sub></i>/( )2 (2<i>n n</i>1)<i>x</i>2<i>n</i> 1(<i>x</i>1)   0 <i>x</i> <sub></sub> 1, 0<sub></sub>


0,5


Do vậy <i>g x<sub>n</sub></i>( ) đồng biến trong <sub></sub>1, 0<sub></sub>. Mặt khác <i>g<sub>n</sub></i>(0)10,


   


( 1) 4 n 0


<i>n</i>


<i>g</i> suy ra <i>g x<sub>n</sub></i>( )0 có nghiệm duy nhất


( 1, 0)



<i>n</i>


<i>x</i>   và <i>g x<sub>n</sub></i>( )0   <i>x</i> ( 1,<i>x<sub>n</sub></i>),<i>g x<sub>n</sub></i>( )0  <i>x</i> (<i>x<sub>n</sub></i>, 0)


0,5


Do vậy /<sub>( )</sub> <sub>0</sub>


<i>n</i>


<i>f</i> <i>x</i>  có nghiệm duy nhất <i>x<sub>n</sub></i> ( 1, 0) và


/<sub>( )</sub> <sub>0</sub> <sub>( 1,</sub> <sub>),</sub> /<sub>( )</sub> <sub>0</sub> <sub>(</sub> <sub>, 0)</sub>


<i>n</i> <i>n</i> <i>n</i> <i>n</i>


<i>f x</i>    <i>x</i> <i>x</i> <i>f x</i>   <i>x</i> <i>x</i>


Từ đó suy ra tồn tại duy nhất <i>x<sub>n</sub></i> ( 1, 0) sao cho


( ) min ( )


<i>n</i> <i>n</i> <i>n</i>


<i>f x</i>  <i>f x</i> với <i>x</i>.


0,5


<b>2) Đặt </b><i>S<sub>n</sub></i>  <i>f x<sub>n</sub></i>

 

<i><sub>n</sub></i>


Do


2 1


1 1 1


( 1, 0) 1


1 1 2


<i>n</i>
<i>n</i>


<i>n</i> <i>n</i>


<i>n</i> <i>n</i>


<i>x</i>


<i>x</i> <i>S</i> <i>n</i>


<i>x</i> <i>x</i>






       



  .


</div>
<span class='text_page_counter'>(49)</span><div class='page_container' data-page=49>

Ta chứng minh không tồn tại 1
2


<i>a</i> sao cho <i>S<sub>n</sub></i>   <i>a n</i> <i>n</i><sub>0</sub>. Thật


vậy giả sử tồn tại 1: <sub>0</sub>


2 <i>n</i>


<i>a</i> <i>S</i> <i>a</i>  <i>n</i> <i>n</i> suy ra






         



2 1


0


1 1


( 1, 0) ( ) ( )


1 2



<i>n</i>


<i>n</i> <i>n</i> <i>n</i>


<i>x</i>


<i>x</i> <i>f x</i> <i>f x</i> <i>a</i> <i>n</i> <i>n</i>


<i>x</i>






  



2 1


1 1


lim


1 2


<i>n</i>


<i>x</i>


<i>a</i>


<i>x</i>


0,5


     




1 1


( 1, 0)


1 <i>x</i> <i>a</i> 2 <i>x</i>





   



1


1 1 1


lim


2 <i>x</i> 1 <i><sub>x</sub></i> <i>a</i> 2 điều này vô lý


Ta lại có <i>S<sub>n</sub></i><sub></sub><sub>1</sub> min <i>f<sub>n</sub></i><sub></sub><sub>1</sub>( )<i>x</i>  <i>f<sub>n</sub></i><sub></sub><sub>1</sub>( )<i>x<sub>n</sub></i>





<i>x<sub>n</sub></i>2<i>n</i> 1(1<i>x<sub>n</sub></i>) <i>f x<sub>n</sub></i>( <i><sub>n</sub></i>) <i>f x<sub>n</sub></i>( <i><sub>n</sub></i>)<i>S<sub>n</sub></i>


Vậy <i>S<sub>n</sub></i>là dãy đơn điệu không tăng bị chặn dưới bởi


1 1


limS


2   <i>n</i><i>S</i> 2


0,5


Nếu 1


2


<i>S</i> chọn 1( 1)


2 2


<i>d</i> <i>S</i>


Do <i>S<sub>n</sub></i> <i>S</i> và limS<i><sub>n</sub></i><i>S</i> <i>n</i><sub>0</sub> : <i>S<sub>n</sub></i><i>S</i>   <i>d n n</i><sub>0</sub>
  1( 1) 1  <sub>0</sub>


2 2 2


<i>n</i>



<i>S</i> <i>S</i> <i>n</i> <i>n</i>


Điều này mâu thuẫn với chứng minh trên vậy limS 1.


2


<i>n</i> <i>S</i>


0,5


<b>4. </b>


</div>
<span class='text_page_counter'>(50)</span><div class='page_container' data-page=50>

Kẻ đường kính <i>AM</i> và <i>AN</i> của đường tròn (<i>O</i>) và (<i>O</i>’).
Gọi <i>I, J, K’</i> là trung điểm của <i>CD, EF, MN.</i>


Vì tứ giác <i>MNCD</i> là hình thang vng nên <i>K’I</i> vng góc <i>CD</i> suy
ra <i>K</i>’ thuộc trung trực của <i>CD</i>


Tương tự <i>K</i>’ thuộc trung trực của <i>EF</i>


Vậy '<i>K</i> <i>K</i>.


2,0


AH cắt đường tròn (O’) tại L.


Chứng minh H đối xứng với L qua PG 1,0


Từ đó suy ra MH song song với NL
Nên <i>MHA</i>900



Vậy H thuộc đường tròn (O) cố định


1,0


<b>5. </b>


<i>(4 đ) </i>


<b>1) Đặt </b><i>P</i><i>x x</i><sub>1 2</sub>....<i>x<sub>n</sub></i><i>x P<sub>i i</sub></i>   <i>P i</i>

1, 2,...,<i>n</i>

và <i>P P</i><sub>1 2</sub>...<i>P<sub>n</sub></i> <i>Pn</i>1




Theo giả thiết




 1  *  1, 2,....,


<i>i</i>
<i>i</i>


<i>x</i> <i>N</i> <i>i</i> <i>n</i>


<i>P</i>







 <sub>1</sub> <sub>2</sub>    *   *


1


1 2


1 1 1 ( 1)


( )( )...( )


<i>n</i>


<i>n</i> <i>n</i>


<i>n</i>


<i>P</i>


<i>x</i> <i>x</i> <i>x</i> <i>N</i> <i>N</i>


<i>P</i> <i>P</i> <i>P</i> <i>P</i>


1,0


Theo giả thiết <i>x<sub>i</sub></i> <i>Q</i> <i>i</i>

1, 2,..., n

<i>P</i> <i>Q</i> <i>P</i> <i>a</i>


<i>b</i>


 



      



với <i>a b</i>, <i>N</i>*,( , )<i>a b</i> 1.


Từ đó suy ra  <sub></sub>   *


1


( )


.


<i>n</i>
<i>n</i>


<i>a b</i>


<i>m</i> <i>N</i>


<i>b a</i>


   


 <i>an</i><i>C a<sub>n</sub></i>1 <i>n</i> 1<i>b</i>.... C <i><sub>n</sub>n</i> 1<i>a b</i>. <i>n</i> 1<i>bn</i> <i>man</i> 1<i>b</i> (1)


</div>
<span class='text_page_counter'>(51)</span><div class='page_container' data-page=51>

Từ đẳng thức (1) ta suy ra
|


|



<i>n</i>
<i>n</i>


<i>a b</i>
<i>b a</i>





 <sub> </sub>


mà ( , )<i>a b</i> 1<i>a</i><i>b</i>1<i>P</i>1
Vậy <i>x x</i><sub>1 2</sub>....<i>x<sub>n</sub></i>1.


<b>2) Ta lại có </b>




1 2


2 .2 ....2 2 .<i>n</i> 2<i>n</i> 2 2 ,<i>i</i> 1, 2,...,


<i>n</i> <i>i</i> <i>i</i>


<i>x</i> <i>x</i> <i>x</i>  <i>y</i>  <i>x</i>    <i>N</i>  <i>i</i> <i>n</i> .
Từ đó suy ra 2 .2 ....2<i>x</i><sub>1</sub> <i>x</i><sub>2</sub> <i>x<sub>n</sub></i> 2<i>n</i> <sub>1</sub><sub>2</sub> ....<i><sub>n</sub></i> <i>n</i> (2)


Đặt




*


1 2


1, 2,....,
1


... 2 (3)


<i>i</i>


<i>i</i> <i>i</i>


<i>n</i>


<i>N</i> <i>i</i> <i>n</i>


<i>n</i>


 


  


   




   



   





Từ suy luận trên ta suy ra số bộ có thứ tự ( ,<i>x x</i><sub>1</sub> <sub>2</sub>,...,<i>x<sub>n</sub></i>)bằng số
nghiệm tự nhiên của phương trình (2) bằng số nghiệm nguyên
dương của phương trình (3).


1,0


Ta goi <i>X</i> là tập các xâu có tính chất độ dài xâu là 3<i>n </i>– 1 bao gồm
2<i>n</i> số 1, <i>n – </i>1 số 0 sao cho số 0 không đứng ở đầu xâu,cuối xâu
và khơng có hai số 0 nào đứng cạnh nhau trong mỗi xâu đó.Gọi
T là tập nghiệm nguyên dương của phương trình (3).Xét ánh xạ


:


<i>f T</i><i>X</i>sao cho nghiệm ( <sub>1</sub>, <sub>2</sub>,....,<i><sub>n</sub></i>)được đặt tương ứng
với xâu (1,…,1,0,1,…,1,0,…..,0,1,…,1) với số các số 1 trong xâu
lần lượt là  <sub>1</sub>, <sub>2</sub>,...,<i><sub>n</sub></i>.Dễ thấy <i>f</i> là song ánh nên<i>T</i>  <i>X</i> .Mà
số phần tử của tập X bằng số cách chọn n-1 vị trí trong 2n – 1 vị
trí giữa các số 1 để đặt số 0,số cách chọn này là <i>C</i><sub>2</sub><i>n<sub>n</sub></i>1<sub>1</sub>.


Vậy số bộ số thỏa mãn yêu cầu bài toán là <i>C</i><sub>2</sub><i>n<sub>n</sub></i>1<sub>1</sub>


 .


1,0



<b> Chú ý: </b>


<i>Học sinh làm theo cách khác nhưng đúng vẫn cho điểm tối đa. </i>


</div>

<!--links-->

×